Сохранен 516
https://2ch.hk/spc/res/115935.html
24 декабря Архивач восстановлен после серьёзной аварии. К сожалению, значительная часть сохранённых изображений и видео была потеряна. Подробности случившегося. Мы призываем всех неравнодушных помочь нам с восстановлением утраченного контента!

Тред тупых вопросов. №15

 Аноним Чтв 03 Июл 2014 01:23:02 #1 №115935 
1404336182473.jpg

Тред гениальных вопросов о жизни, вселенной и всего такого!

Спрашиваем то, за что в других местах выдают путёвку в биореактор. Здесь анонимные ученые мирового уровня и просто любители критически рассмотрят любые гениальные идеи и нарисованные в Paint схемы, ответят на смелые рассуждения будущих светил космонавтики, вступят друг с другом в околонаучный диспут.

Прошлый тред тут:

Аноним Чтв 03 Июл 2014 04:15:36 #2 №115944 

В чём смысл бытия?

Аноним Чтв 03 Июл 2014 04:37:00 #3 №115945 

>>115944
прост))

Аноним Чтв 03 Июл 2014 04:51:18 #4 №115946 

>>115944
>В чём смысл бытия?
При чем здесь астрономия и космонавтика, любезный?
Вам с такими вопросами в /re/ лучше обратиться.
>>115945
>прост))
Вполне себе ответ, кстати.

Аноним Чтв 03 Июл 2014 11:55:27 #5 №115955 

Какой будет пилотируемая космонавтика, когда спишут МКС? Есть ли реальные планы строительства новых станций, и что это будут за станции - очередные клоны "Салюта", или какой-то йоба-некстген?

Аноним Чтв 03 Июл 2014 12:02:15 #6 №115956 

>>115955
не будет новых станций после МКС. "нет задач"
максимум временные миссии по сборке сложных телескопов и другого оборудования типа ATLAS
может облёты луны и марса лет через 20
и на крайняк туризм на драгонах

Аноним Чтв 03 Июл 2014 12:10:21 #7 №115957 

>>115956
> не будет новых станций после МКС. "нет задач"
А как же китайцы и бигелоу?

Аноним Чтв 03 Июл 2014 12:19:24 #8 №115959 

>>115957
Но они же инопланетяне. Никого не ебёт, что там у них, есть ли станции или нет. Может, они уже на Марсе давно город построили и сферу Дайсона начали проектировать, всем наплевать.

Аноним Чтв 03 Июл 2014 12:42:45 #9 №115964 

>>115959
Они лунный зонд не могли посадить куда надо, какие инопланетяне, о чем ты.

Аноним Чтв 03 Июл 2014 12:44:55 #10 №115965 

>>115957
У китайцев планируется нечто Миро-подобное, чтобы самостоятельно отработать опыт по сборке и поддержанию модульных станций, давно полученный СССР и США. Учебная конструкция, не более. Бигелоу имеет все шансы обосраться, т.к. зависит от интереса туристов. Посмотрим.

Аноним Чтв 03 Июл 2014 12:48:40 #11 №115967 

>>115956
>нет задач
Ну не то что бы нет, но как-то дороговато выходит. Может каждый, кому надо, запустят по паре сцепленных бочек подешевле, что бы дальше салат и мух выращивать.
http://www.mcc.rsa.ru/plan.htm

Аноним Чтв 03 Июл 2014 16:29:43 #12 №115997 
1404390583384.jpg

>>115935
Почему Марс - красная планета?
Читал про ржавчину и все дела, но у меня с телескопа он нихуя не красный. Скорее охра с оттенками голубого. Да и в зоге видал, что НАСА фильтры подставляет.

Аноним Чтв 03 Июл 2014 16:43:13 #13 №116000 

>>115997
какого нахрен телескопа?
он у тебя там звёздочкой сиять должен
может ты нибиру случайно разглядываешь? за тобой уже выехали

Аноним Чтв 03 Июл 2014 16:53:13 #14 №116002 

>>115997
>Почему Марс - красная планета? у меня с телескопа он нихуя не красный
Потому что без телескопа она все же скорее красноватая, чем желтая. В телескоп люди смотрят лет 400, а невооруженным глазом гораздо дольше.
>с оттенками голубого.
У тебя хроматические аберрации
>Да и в зоге видал
Я вчера снежного человека в центре ДС2 видел. Ща в ЗОГе напишу.
>НАСА фильтры подставляет.
Ну подставляет, что с того-то? Это делается в научных целях.
Если ты про баланс белого, то камера и глаз воспринимают цвет по-разному, тем более, в условиях марсианской атмосферы. Так что надо подкручивать, что бы сделать снимок близким к тому, что увидел бы глаз, а не камера.

Аноним Чтв 03 Июл 2014 17:37:57 #15 №116011 

Почему в спейсаче столько раковых тредов расплодилось, а ламповых обсуждений хотя-бы экзопланет нету? Охуенная ведь тема посоны.

Аноним Чтв 03 Июл 2014 17:43:20 #16 №116014 

>>116011
Потому что ты не уходишь не создашь никак.

Аноним Чтв 03 Июл 2014 17:46:54 #17 №116015 

>>116014
Так типа создать шоле? Там будет кто-то сидеть?

Аноним Чтв 03 Июл 2014 17:47:41 #18 №116016 

>>116011
>а ламповых обсуждений хотя-бы экзопланет нету?
Создай тред, обсудим. Почему все думают, что за них кто-то что-то должен делать?

Аноним Чтв 03 Июл 2014 17:48:49 #19 №116017 

>>116015
Заодно и проверишь. Это же без регистрации, бесплатно и без СМС.

Аноним Чтв 03 Июл 2014 17:59:30 #20 №116019 

>>116017
>>116016
http://2ch.hk/spc/res/116018.html хуле

Аноним Чтв 03 Июл 2014 18:16:35 #21 №116022 

>>116019
Вот молодец, не сложно же?

Аноним Чтв 03 Июл 2014 18:21:22 #22 №116023 

>>116022
Таки нет. посмотрим похуй всем или нет. Постараюсь регулярно вбрасывать чего.

sageАноним Чтв 03 Июл 2014 18:25:40 #23 №116025 

>>116023
>похуй всем
Ты знал.

Аноним Чтв 03 Июл 2014 18:28:38 #24 №116026 

>>116025
Ну и ладно, вдруг кто забредёт. В любом случае это лучше ваших майнкравтов и огурцов и ближе к тематеке. Олсоу, сажу схорони для тредов вроде http://2ch.hk/spc/res/115929.html

Аноним Чтв 03 Июл 2014 18:51:11 #25 №116028 

поясните про палку и скорость звука в ней из прошлого треда.
если палку длиной 100000км вращать со скоростью 1 оборот в секунду то на её концах скорость будет скорость света?
вроде 2*пи*R=L = 314000 км за 1 оборт в секунду. Где подвох?

Аноним Чтв 03 Июл 2014 19:08:03 #26 №116036 

>>116028
парадокс эренфест

/вопрос

Аноним Чтв 03 Июл 2014 19:27:42 #27 №116040 
1404401262058.jpg

>>116028
Особо не читал тот унылый разговор из прошлого треда и не считал, но не забывай, что число пи по ОТО может меняться, и причем весьма нехило.

Аноним Чтв 03 Июл 2014 22:06:00 #28 №116057 

Программа "Marsone" взлетит или нет? Вроде людей тренировать уже должны, да и по сути они смертниками являются, лол.

Аноним Чтв 03 Июл 2014 22:08:29 #29 №116058 

>>116057
>взлетит или нет?

Сомневаюсь я. Набрали каких-то упырей, а надо было набрать денег сначала.

Аноним Чтв 03 Июл 2014 22:31:36 #30 №116061 

>>116057
Чтобы полетел к 2020 году нужно было начать НИОКРы лет 10 тому назад. Так что...

Продолжая тему путешествия на околосветовых скоростях из предыдущего треда. Защитит ли летящий перед кораблем астеройд от поджаривания из-за излучений?
Довольно быстро любой астероид сгорит, как быть тогда?

Аноним Чтв 03 Июл 2014 23:26:40 #31 №116064 

>>116057
Явно не взлетит. Сплошное беспруфное кукареканье и отсутствие фактически всего (кроме разве что рекламной кампании).
>>116061
Астероид - да (ну смотря какой). Только его тоже разогнать надо. А до околосветовых скоростей ты еще корабль попробуй разгони. Так что уж будем по старинке, на парусах, ЭРД и ЯРД пыхтеть на сотых и тысячных долях световой до ближайших звезд, другого не планируется.

Аноним Чтв 03 Июл 2014 23:29:10 #32 №116066 

>>116064
>другого не планируется.

ПУЗЫРЬ АЛЬБУКЕРКЕ

Аноним Чтв 03 Июл 2014 23:33:09 #33 №116067 

>>116057
Очевидная клоунада. Здесь есть где-то тред про неё.

Аноним Чтв 03 Июл 2014 23:44:29 #34 №116073 

Почему бы не создать машину времени? Берем огромное зеркало диаметром пару световых лет, отодвигаем его на большое расстояние ~1000 с.л., направляем на Землю. PROFIT! Смотрим в телескопы на это зеркало и наблюдаем историю своими глазами.

Аноним Чтв 03 Июл 2014 23:47:59 #35 №116076 

>>116073
>Смотрим в телескопы на это зеркало и наблюдаем историю своими глазами.
Зачем? Тебе ютуба мало что ли? Проще все записывать и снимать, хранить в архивах чем двигать зеркала.

Аноним Птн 04 Июл 2014 05:36:35 #36 №116105 

>>116073
зачем смотреть в телескоп? можно на месте глянуть

Существует какой то план для космонавтов при начале тотального пиздеца на планете? Сколько они смогут автономно прожить? И могут ли наложить на себя руки при желании?

Аноним Птн 04 Июл 2014 06:14:15 #37 №116106 

>>116105
При начале тотального пиздеца на планете будет не до космонавтов.

> Сколько они смогут автономно прожить?
Смотря где. На МКС - месяцами. В транспортных кораблях - недолго.

> И могут ли наложить на себя руки при желании?
Кто-то из экипажа аварийного Союза-33 кажется обмолвился, что посещали мысли об открытии клапана, если вдруг останется на орбите куковать дольше чем хватит кислорода.

Аноним Птн 04 Июл 2014 06:41:42 #38 №116107 

>>116106
>На МКС - месяцами.
вот про это, только поподробнее
есть ли какие то методочки, параграфы, указания и т.п. это же все просчитывалось. Например если нет возможность забрать космонавтом в орбиты (нет возможность запустить союзы на орбиту и спускаемые аппараты на орбите повреждены)
или это все секретно?

Аноним Птн 04 Июл 2014 07:04:58 #39 №116108 

>>116107
Ну не месяцами, но резерв там порядочный. Там целый комплекс СЖО, называется ECLSS и разбросан по модулям станции. Там и воздухо-очистители и регенераторы (ARS, OGS, ROS, Электрон, Вика, Воздух и т.п.), поддержание климата, очищение воды, запасы жрачки, кислорода в твердом виде и т.п.

> или это все секретно?
Лол, нет конечно. Все что происходит на МКС, публично документировано до мелочей, луркай.

Аноним Птн 04 Июл 2014 11:21:17 #40 №116115 

>>116105
В случае любой угрозы пиздеца (например, столкновения с космическим мусором) космонавты на МКС немедленно прячутся в пристыкованных кораблях. Вернуться с орбиты - не проблема в любой момент, даже расчетов сложных не надо, лишь легкий ретроградный пердеж. Другое дело - в этом случае место падения будет практически случайным - ладно еще в тайге, так можно ведь и посреди океана.
Но это конечно маловероятно - уж хватит у них запасов, чтобы понаворачивать кругов вокруг Земли и выбрать что-нибудь удобное.

Аноним Птн 04 Июл 2014 11:53:10 #41 №116116 

>>116115
>и выбрать что-нибудь удобное
я бы в америкашку свалил при таком случае

Аноним Птн 04 Июл 2014 12:48:30 #42 №116126 
1404463710556.png

Поясните за эту гениальную йобу, светило разума дедов. Как оно вообще на орбиту что-то вывело, там же вроде как нужно сначала на суборбитальную выйти, движки потушить, а потом в апогее допиздячить до первой космической. А эта йоба-то врубала и первую и вторую ступени одновременно, потом первая (ну боковые блоки кароч) отстреливались а вторая продолжала ебашить, и под конец вторая ступень со спутником оказывалась на орбите. Это как вообще? Движки вроде ж как не тушились и заново не зажигались.

Аноним Птн 04 Июл 2014 13:02:18 #43 №116131 

>>116126
> там же вроде как нужно сначала на суборбитальную выйти, движки потушить, а потом в апогее допиздячить до первой космической.
В KSP переиграл? Так у ракеты не шибко большой ТВР, чтобы можно было движки выключать.

Аноним Птн 04 Июл 2014 13:09:40 #44 №116133 

>>116131
Ну, учитывая что ТВР у нее был около 1.2, поднималась она и правда реально медленно. Я так понимаю, орбита у спутника была дико вытянута именно потому, что движки не вырубались?

sageАноним Птн 04 Июл 2014 13:15:32 #45 №116134 

>>116126
В чем проблема-то? Горючки больше в центральном блоке, поэтому он летит дальше, при этом тяговооруженности уже достаточно на этом участке траектории, да ещё и ступень облегчена уже. Тяга рассчитана так, чтобы топливо кончилось по достижении расчетной орбиты.

> там же вроде как нужно сначала на суборбитальную выйти, движки потушить, а потом в апогее допиздячить до первой космической
Это у тебя в KSP так. На практике выгодней траекторию делать достаточно пологой, чтобы постоянно пиздячить. (кстати в KSP это особенно большую экономию даёт, т.к. там движки нереалистично тяжелые). В реальных РН паузы между бёрнами если и есть, то небольшие.

Блять, почему вы упорно не хотите пилить новые треды? Нахуя вы пишете нормальные в общем-то вопросы в этот?

sageАноним Птн 04 Июл 2014 13:17:54 #46 №116135 

>>116133
> Я так понимаю, орбита у спутника была дико вытянута именно потому, что движки не вырубались?
И да и нет. Круговую тоже так делают.

Аноним Птн 04 Июл 2014 13:21:52 #47 №116138 

>>116134
А что там с пологостью траектории-то? Как вообще gravity turn-то проходит? Просто я недавно RSS обмазался мод на реалистичную солнечную систему для ксп так там советуют начинать поворачивать на 5 градусов при достижении ракетой скорости 100 м/с. Неужто так и в реале?

>>116135
Это ж чертова куча расчетов нужна, для такой-то точности, чтобы прям и круговая, и на нужной высоте, и все это - не вырубая двигатели. Так эффективнее, что ли? Типа если вырубить двигло - будут гравитационные потери больше?

Аноним Птн 04 Июл 2014 13:32:25 #48 №116140 

>>116138
> Это ж чертова куча расчетов нужна,
welcome to real life!

> Неужто так и в реале?
Посмотри камеру с борта любой РН, и вопросы отпадут.

Аноним Птн 04 Июл 2014 13:35:19 #49 №116142 

>>115935
Какой список тематической литературы стоит прочесть? Собираюсь сначала пройти полностью физику за школу (это 5-11 классы), так как когда был в ней, то всегда просто заучивал, и не понимал ничего, а затем перейти в астрофизику

Аноним Птн 04 Июл 2014 14:15:17 #50 №116151 

>>116138
>чтобы прям и круговая, и на нужной высоте, и все это - не вырубая двигатели
В КСП тоже так можно. Двигатели не вырубают в том числе и потому, что хуй знает включатся ли они обратно. А некоторые и не включатся, потому как для этого не предназначены.

Аноним Птн 04 Июл 2014 21:49:20 #51 №116185 
1404496160098.png


>>116126
> там же вроде как нужно сначала на суборбитальную выйти, движки потушить, а потом в апогее допиздячить до первой космической.
Это называется «двухимпульсное выведение». Применяется когда надо вывести нагрузку сразу на высокую орбиту и хочется сэкономить топливо самого КА.

>>116134
В огуречнике просто кисель вместо атмосферы и стоковая аэродинамика говённая.

>>116138
> так там советуют начинать поворачивать на 5 градусов при достижении ракетой скорости 100 м/с. Неужто так и в реале?
Ты не поверишь… (пикрелейтед из Proton Launch System Mission Planner's Guide)

> Это ж чертова куча расчетов нужна, для такой-то точности, чтобы прям и круговая, и на нужной высоте, и все это - не вырубая двигатели.
Ато!.жпг «Траектория полёта была рассчитана под руководством М. В. Келдыша в Математическом институте им. В. А. Стеклова».(ц) (Это про «Луну-3». Она выводилась сразу на траекторию к Луне, без использования промежуточной околоземной орбиты).

Аноним Суб 05 Июл 2014 05:39:24 #52 №116212 

Почему люди так хотят полететь на Марс? Там же холодная безжизненная пустыня, а изучить его вдоль и поперек можно с помощью зондов и марсоходов. Зачем вообще посылать туда людей?

Аноним Суб 05 Июл 2014 06:07:38 #53 №116214 

>>116212 -> >>115945

Аноним Суб 05 Июл 2014 08:49:36 #54 №116215 

>>116212
Потому что это просто охуенно.

Аноним Суб 05 Июл 2014 08:52:09 #55 №116216 

>>116212
> люди так хотят
Как так? Какие люди? Некоторые люди хотят убить всех человеков или спасти мир, например.
хотят они бледь. Ну да, хотят. "Было бы неплохо погулять по марксу и - домой". А как поработать ради этого лишний час три раза в неделю - так сразу "Нам и здесь не плохо".
Короче тех кто хочет на маркс было 200тыщ. Остальным семи миллиардам либо похуй, либо реально не до того. Один из 35 тыщ - хочет. Считай не хочет никто.

Аноним Суб 05 Июл 2014 11:41:09 #56 №116222 

>>115935
Поездните за минимальные размеры стыковочных узлов.

Поцчему бы не вывести на орбиту за стопицот раз миллиард пикоспутников, и чтоб они сделали из себя верфь для космических кораблей? Хотя бы каркас?

sageАноним Суб 05 Июл 2014 11:47:16 #57 №116224 

>>116222
Ебли много.

Аноним Суб 05 Июл 2014 12:15:08 #58 №116228 

>>116222
Во первых — заебёщься, как уже сказал >>116224. Во вторых: вот выводим мы через несколько лет последнюю деталь и тут внезапно оказывается что часть блоков (которые выводили первыми) уже выработали весь ресурс, а ещё часть ресурс не выработала, но всё равно не работает (ибо теория вероятности, MTBF, гроб, кладбище, пидор). И всё надо начинать заново.

Аноним Суб 05 Июл 2014 12:59:16 #59 №116230 

Почему принято думать что долетев до ближайших звёзд мы тут же встретим пригодные для жизни джунгли и туземцев с копьями? Мы ведь сейчас можем доставить человека на любую планету солнечной системы, но не делаем этого потому что ему там нечего делать. Так же и с экзопланетами - смысл куда то лететь чтобы найти там такие же мертвые камни и лед, как и в нашей системе.

Аноним Суб 05 Июл 2014 13:33:01 #60 №116232 

>>116230
>принято думать
Кем?
>смысл куда то лететь чтобы найти там такие же мертвые камни и лед
Когда появится возможность туда слетать, то можно слетать хотя бы за теми же камнями и льдом.

Аноним Суб 05 Июл 2014 14:06:21 #61 №116235 

>>116228
Ну дак сделать каркас поменьше, как их лего. А спутники как можно более простыми, и штамповать их на конвейере.

Аноним Суб 05 Июл 2014 16:17:51 #62 №116245 

>>116185
>огуречнике просто кисель вместо атмосферы

В Орбитёре та же херь, дельтаглайдер\шаттл-а больше 700м\с не разгоняется.

Аноним Суб 05 Июл 2014 21:15:31 #63 №116284 
1404580531830.jpg

>>116245
> 700м\с
Кек. Это же 2500км/ч, больше двух махов. Попробуй ИРЛ у земли так разогнаться, я бы посмотрел на это. В огурцах в ванильной аэродинамике сопротивление ещё больше, об этом речь.

Аноним Суб 05 Июл 2014 21:22:00 #64 №116287 
1404580920810.jpg

>>116222
> Поцчему бы не вывести на орбиту за стопицот раз миллиард пикоспутников, и чтоб они сделали из себя верфь для космических кораблей?
И как пикоспутники будут подруливать друг к другу? И нахуй нужны эти сложности, когда можно выводить цельные корабли и лепить из них йобу любых размеров? Так уже и делается, собственно.
> Хотя бы каркас?
Но зачем, когда можно выводить большими кусками? И ещё будет болтаться как в кербале. Станция и так динамически очень сложная конструкция. Почитай что такое Integrated Truss Structure и сколько у неё функций.

Аноним Вск 06 Июл 2014 00:49:18 #65 №116304 

>>116284
Гиперзвуковые ракеты это с легкостью делают. А вот в ксп даже если разогнать какой-либо объект до околосветовых скоростей и уронить в атмосферу, то он в ней сразу же завязнет.

Аноним Вск 06 Июл 2014 01:01:27 #66 №116306 

>>116304
В Орбитере я впервые в жизни наглядно осознал что такое скорость света - разогнал редактором дельта глайдер. Оказывается это не так уж и быстро.

Аноним Вск 06 Июл 2014 10:20:02 #67 №116322 

>>116306
Я это в детстве осознал, когда узнал, что до солнца 8 минут. И какие расстояния вообще между звездами и галактиками. Предельная скорость невероятно мала.

Аноним Вск 06 Июл 2014 20:03:49 #68 №116360 
1404662629887.jpg

>>116212
На данный момент люди продолжают быть несколько более универсальными инструментами, нежели аппаратура. Кроме того - время прохождения сигнала (скорость света-то ограничена, как это ни печально). Если на Луне это почти незаметно, то на Марсе оно здорово мешает связи в реальном времени с марсоходами и сжирает кучу рабочего времени (скоростные рекорды, как у Луноходов - не поставишь). Ну и кроме того - отработка колонизации, авось Илон наш Маск таки запилит чудовищными усилиями через пару-тройку десятков лет свою хваленую колонию на 80.000 рыл.
>>116230
Не, ну серьезно, кем принято думать? Данные наблюдений (надо не забывать, что мы ищем потерянные часы в куче говна под фонарем, как в анекдоте) показывают нам пиздецовые горячие юпитеры у красных карликов, которые нам совершенно неинтересны. Ну это ничего - еще пара десятков лет наблюдений (а уж прошедшие пара десятков лет наблюдений, надо заметить, были пиздец как интересны!) и увидим самые что ни на есть землеподобные планеты в зоне Златовласки у звезд, похожих на Солнце, да и еще и в десятке-другом световых лет.

Аноним Вск 06 Июл 2014 21:51:31 #69 №116364 

>>116360
Ну как же, открываешь любую статью про экзопланеты и видишь там нарисованные художником моря, леса и задумчивый аборигенов. Читаешь любую фантастическую книгу или фильм или игру - везде та же бадяга. Более менее проработанные проекты межзвездных перелетов все так же основаны на молчаливом предположении что колонистам есть куда лететь.
>землеподобные
Таки ведь и Венеру с Марсом в землеподобные засчитывают. Приличные такие планеты в зоне жизни, хоть завтра прилетай и селись - если рассматривать их с десятка световых лет, то ведь хрен определишь что это филиалы ада в нашем бренном мире. И никто нас там не ждёт, и вообще - скорей всего человек без защитных приспособлений не сможет выжить ни на одной другой планете вселенной, потому что он приспособлен именно под нашу планету и наш вариант биосферы.

Аноним Вск 06 Июл 2014 22:02:13 #70 №116368 

>>116364
По-моему ты ерунду пишешь. Нет такого заблуждения, я его не замечаю, твои выводы для меня даже звучат странно. При чем тут фантастика, кстати? Там вот гиперпространство везде используют, машины времени, звездный десант, надо ли это великолепие тоже на наши технологии и быт спроецировать?

Аноним Вск 06 Июл 2014 22:21:16 #71 №116373 

>>116368
Не могу утверждать что это заблуждение, потому что я не знаю правды. Я замечают лишь непроговариваемую вслух уверенность что нам есть куда лететь и что проблема лишь в способе доставки человеческой тушки в разумные сроки.

Аноним Вск 06 Июл 2014 22:22:15 #72 №116374 

>>116373
Нет такого. Тебе кажется. Выпей водки, товарищ, всё пройдет.

Аноним Вск 06 Июл 2014 23:06:16 #73 №116376 
1404673576001.jpg

>>116364
В статьях про экзопланеты рисуют предполагаемый вид на основании имеющихся данных и фантазии художника (которая позволяет трактовать весьма скудные имеющиеся данные весьма широко).
Про фантастов - ну тут и говорить нечего, я и сам любитель почитать Стругацких или там Буджолд, но это ну никак не надо связывать с реальным исследованием космоса.
По жизнепригодности планет - так тот же Марс в лучших своих местах (долины Маринера) немногим менее жизнепригоден, чем внутренние районы Антарктиды, а там полярники вполне успешно работают. Уникальность Земли - хорошая конечно гипотеза, но уж больно частыми во вселенной оказываются всякие уникальные условия (вроде гигантского столкновения), чтобы Земля и прямо была уникальной.
Ну и насчет, в конце концов, рассмотрения с десятка световых лет - экзопланеты исследуют совсем немного, эта наука практически в зачаточном состоянии (нынешние инструменты под их исследование весьма примитивны, даже тот же хваленый Кеплер), и явно позитивным результатом можно считать то, что таки нашлись планеты, более-менее подходящие под критерии жизнепригодности (ну и наконец добавлю, что тот же Марс миллиард-другой лет назад был вполне пригоден для заселения колонистами, оборудованными кислородной маской, тулупом и валенками).

Аноним Пнд 07 Июл 2014 00:11:30 #74 №116385 

>>116376
>Уникальность Земли - хорошая конечно гипотеза, но уж больно частыми во вселенной оказываются всякие уникальные условия
Ну хуй знает, что сама гипотеза, что противные ей - две крайности, как по мне. Если конкретно отдельные события и не являются редкостью, то сам набор этих событий, позволивший появится разумной жизни на Земли - действительно очень редкое стечение обстоятельств. Считаю, что количество планет в галактике, на поверхность которых человек может выйти без последствий для здоровья хотя бы на час, можно по пальцам пересчитать.

Аноним Пнд 07 Июл 2014 00:20:52 #75 №116386 

>>116376
Антарктида по сравнению с Марсом санаторий. Там есть нормальная сила тяжести, есть магнитное поле, плотная атмосфера из азота и кислорода, есть вода, есть полгода стабильного солнечного света, есть действующие вулканы, есть гребаная куча полезных ископаемых.

Аноним Пнд 07 Июл 2014 00:29:33 #76 №116389 
1404678573262.jpg

>>116364
> Ну как же, открываешь любую статью про экзопланеты и видишь там нарисованные художником моря, леса и задумчивый аборигенов.
И где именно ты открываешь эту статью? В журнале из списка ВАК или, в лучшем случае, какой-нибудь «Популярной механике» а то и вообще «Технике молодёжи»?

>Читаешь любую фантастическую книгу или фильм или игру
А тут то что удивительного? Ясен пень, что для развлечения люди будут охотнее покупать космооперу про синих красоток нетяжёлого поведения, чем производственный роман о жизни космонавтов и учёных, бьющихся над загадками «горячего Юпитера».

> Более менее проработанные проекты межзвездных перелетов все так же основаны на молчаливом предположении что колонистам есть куда лететь.
> колонистам есть куда лететь
> колонистам.
Ну ты понел. Вопрос из серии «Почему, если ехать по железной дороге, то обязательно попадёшь в какой-нибудь город?».

Аноним Пнд 07 Июл 2014 06:08:01 #77 №116394 

>>116386
Антарктиду не колонизируют, потому что:
а)Государства не могут договориться, кому она достанется, и потому лучше уж, чтоб она была ничейной
б)Она, сука, маленькая, и у нас на планете такая одна. Если ее засрать, то будет типа жалко
в)Она находится слишком близко, а это значит, что никакой реальной защиты от БП она не предоставляет.

А вот если бы на месте Марса была планета с условиями уровня Антарктиды, то практически наверняка там уже сейчас были бы постоянные поселения.

Аноним Пнд 07 Июл 2014 09:14:05 #78 №116402 

>>116386
> есть гребаная куча полезных ископаемых.
Так фига ли бы их на Марсе не было? Из одного пылевого облака сконденсировались же. Ну кроме угля с нефтегазом конечно, да и нафиг они нужны.

Аноним Пнд 07 Июл 2014 11:59:18 #79 №116422 

>>116402
На Марсе нет осадочных пород.

Аноним Пнд 07 Июл 2014 12:05:43 #80 №116424 
1404720343300.jpg

>>116422
У меня для тебя ломающие новости.

Аноним Пнд 07 Июл 2014 13:23:14 #81 №116431 

варп двигатель возможен?
или фантастика пока? хотя бы теории есть?

Аноним Пнд 07 Июл 2014 13:27:26 #82 №116432 

>>116431
>хотя бы теории есть?
http://ru.wikipedia.org/wiki/Пузырь_Алькубьерре

Аноним Пнд 07 Июл 2014 14:05:49 #83 №116435 
1404727549291.png

>>116432
Поясните платину про Пузырь. Если пространство сжать, то и скорости в нем "сожмутся" ведь, сам корабль сожмется и никакого профита не будет. Само по себе выражение "космический корабль может двигаться, сжимая пространство перед собой и расширяя его позади" на эти мысли наталкивает. Как же на самом деле эта концепция работает? Вот смотрите на нарисованную в Paint схему.

Расстояние между линиями, допустим, фиксировано, скажем, равно световому году. Через шесть лет оба корабля оказываются на верхней полоске, но "1" летел по обычному пространству, а "2" сперва по расширенному, потом по сжатому и потом снова по расширенному. При этом для него по сути ничего не менялось кроме масштабов, расстояние в световой год он преодолевал за то же время, за которое преодолевал это расстояние в любое другое время, вне зависимости от пространства. Просто в сжатом он для наблюдателя этой картинки летел медленно, а в расширенном быстрее. Для него он летел всегда с одной скоростью.

Мне-то понятно, что концепция работает как-то иначе, но как? Она каким-то образом сжимает пространство, а корабль пронзает его в своем измерении?

Аноним Пнд 07 Июл 2014 14:22:17 #84 №116436 

>>116435
>Поясните платину про Пузырь.

Лоуренс Максвелл Краусс поясняет по хардкору.

Аноним Пнд 07 Июл 2014 14:43:54 #85 №116441 

>>116435
> Если пространство сжать, то и скорости в нем "сожмутся" ведь, сам корабль сожмется и никакого профита не будет.
В том-то и дело, что на корабль сжатие не распространится. Корабль как бы вне пространства. Оно сжимается только для корабля, поэтому для пространства корабль растягивается.

Аноним Пнд 07 Июл 2014 14:57:27 #86 №116444 
1404730647548.png

>>116435

Возьми ковер на полу. Сними со стены блеать и поставь на его краю батл пива. Сядь на другую сторону. Твоя задача, не двигая жопу добраться до пива. Теперь двигай ковер к себе, складывая его. Вот тебе принцип действия пузыря. Довольно по площади большой ковер можно сжать в небольшой рулончик. Точно так же этот ковер можно развернуть у себя за спиной, опять таки не отрывая передвигая жопу.

Аноним Пнд 07 Июл 2014 16:56:36 #87 №116458 

>>116444
>Твоя задача, не двигая жопу добраться до пива
всегда так делаю. И не знал что такие технологии использую

Аноним Пнд 07 Июл 2014 19:09:57 #88 №116479 

>>116435

Если ты про пузырь альбукерке, то они хотят сделать ctrl+x ctrl+v на некое пространство. Возможно даже с кораблем, возможно даже из корабля.

Аноним Пнд 07 Июл 2014 19:49:47 #89 №116483 

>>116360
>ищем потерянные часы в куче говна под фонарем, как в анекдот
Что за анекдот?
>>116444
Ну достаёшь ты пиво, а разве оно само не должно сжаться при этом?

Аноним Пнд 07 Июл 2014 20:00:53 #90 №116485 

>>116483
>Ну достаёшь ты пиво, а разве оно само не должно сжаться при этом?
Ну вообще это образный пример. Ты подвинул пространство, пиво перед тобой, разжимаешь пространство за собой, пиво остается рядом.

Аноним Пнд 07 Июл 2014 23:09:16 #91 №116511 

>>116485
Но если нам нужно добраться до пива, проскользив рукой по поверхности собранного в складки ковра, то проделаем мы прежний путь. Вот этот момент и смущает.

Видимо, концепция пузыря одновременно предполагает, что мы эти складки заодно и "пронзаем" каким-либо образом.

Аноним Пнд 07 Июл 2014 23:36:13 #92 №116513 
1404761773987.jpg

Где найти аналог >>116185-пика для пикрелейтеда? Собрал его в обмазанном модами на фулл реализм земля 1:1, реалистичное топливо, реалистичные движки, менее говеная аэродинамика, запускаю соответственно с Байконура КСП, орбита получается хуевая потому что никак не могу понять, под каким углом и когда проводить гравитационный поворот. И еще, у орбитальной ступени центр ракеты после отсоединения боковых бустеров, в нем где-то 50% топлива остается дикая, нечеловеческая тяговооруженность. Это вообще нормально? На орбиту ж вроде как меееедленно выходят, а эта йоба настолько мощная что свой крохотный легонький спутник запиздячивает с невероятным ускорением.

Аноним Втр 08 Июл 2014 00:36:40 #93 №116525 

>>116513
По реализму угорать куда лучше в Орбитёре. и уж точно не в тупотреде
http://www.pin-plus.ca/orbiter/addon.php?id=10

> Это вообще нормально?
Тяга дросселируется жи. Вообще, чтобы о чем-то говорить, нужен симулятор, а не обмазанный модами KSP.

Аноним Втр 08 Июл 2014 00:40:34 #94 №116526 

>>116511
Ты ведь робот? Ты не можешь в образы и абстракции, да?

Аноним Втр 08 Июл 2014 08:03:28 #95 №116543 

>>116511
Да. Пространство сжимается, не сжимая вместе с собой корабль. В этом и заключается смысл пузыря, он "выносит" корабль из пространства.
Поэтому со стороны корабль будет вытянутым.

Аноним Втр 08 Июл 2014 08:20:36 #96 №116544 

>>116513
>Где найти аналог >>116185-пика для пикрелейтеда?
Если тебе именно для двухступенчатой оригинальной семёрки, то спредшит ратмана в зубы и вперёд, на численное решение дифуров. Для современных «Союзов» есть руководства пользователя: http://www.arianespace.com/launch-services-soyuz/Soyuz_Users_Manual_CSG_June06.pdf

>И еще, у орбитальной ступени центр ракеты после отсоединения боковых бустеров, в нем где-то 50% топлива остается дикая, нечеловеческая тяговооруженность. Это вообще нормально?
Ну так ведь тяга (в вакууме) у РД-108 912 килоньютона, а масса заправленной второй ступени (не считая пятитонной термоядерной боеголовки) 93,5 тонны. Естественно, к моменту разделения у второй ступени и будет тяговооружённость в районе 1,5.

Аноним Втр 08 Июл 2014 18:45:58 #97 №116606 

>>115935
Косманы, скажите за накопление радиации в организме, это правда или нет? Например это http://proatom.ru/modules.php?name=News&file=article&sid=5086
Или "накопление радиации" это лишь повышение числа в организме радионуклидов типа цезия или йода какого нибудь?

Аноним Срд 09 Июл 2014 04:24:53 #98 №116669 
1404865493797.jpg

>>116606
Пролистал. "Накопление радиации" является какой-то феерической хуйней. Возможно имеется в виду: 1) таки накопление радионуклидов, 2) наведенная радиация после нейтронного облучения, 3) длительное получение небольших дозняков.
Из всех этих вариантов для космонавтов опасен лишь третий (при наличии минимальной биологической защиты на ихнем РИТЕГе, и отсутствии у них желания расковырять РИТЕГ отверткой или ломом). Ну и так вот - данные Кьюриосити свидетельствуют о том, что полет к Марсу и пребывание на его поверхности даже без биозащиты обеспечивают космонавту весьма приемлимую дозу, не сахар, конечно, но космонавты на МКС, если сильно там задерживаются, получают не особо меньше, и как-то живы в основном. Это если не будет неудачной солнечной вспышки. А если будет - профессия таки опасная, тут только заворачиваться в простыню и ползти в радиационно зашищенный отсек, благо современные радиопротекторы весьма эффективны.

Аноним Срд 09 Июл 2014 05:41:29 #99 №116673 

>>116669
Речь как раз о этих маленьких дозняках, разве они нанесут вред, ведь они не смертельны и не накапливаются по некоторым утверждениям.

Аноним Срд 09 Июл 2014 07:02:54 #100 №116676 

>>116673
Согласно современной беспороговой концепции, накапливаются. Она применяется именно потому, что на практикте накопление дозы отчётливо наблюдается по увеличению вероятности возникновения рака.

Аноним Срд 09 Июл 2014 11:35:07 #101 №116687 

>>116673
Сложный вопрос. Линейная беспороговая концепция не шибко стройна и есть подозрение, что является следствием радиофобии. Ибо - 1) радиационный гормезис 2) большой эффект при сверхмалых дозах из-за отсутствия адаптивного ответа (нередко он и посильнее, чем эффект при существенно больших дозах).
В любом случае - длительное проникающее ионизирующее облучение в любом случае (и при любых дозах) не сахар, как бы ни была хороша адаптация, ибо ДНК и РНК все равно немного ломаются. Другое дело - не стоит его переоценивать.

Аноним Срд 09 Июл 2014 12:41:29 #102 №116691 

>>116687
Ты ведь из тех поехавших, что носятся со своими гениальными гипотезами и кукарекают на каждом углу? Если нет, то приложи пожалуйста пруфы.

Аноним Срд 09 Июл 2014 13:13:17 #103 №116694 

>>116676
>на практикте накопление дозы отчётливо наблюдается по увеличению вероятности возникновения рака.
Только на больших дозах, близких к летальным.

>>116687
>большой эффект при сверхмалых дозах
Вот как раз он и не наблюдается.

Аноним Срд 09 Июл 2014 16:37:08 #104 №116753 

Я так понимаю человек может жить под этими лучами, но предел переносимой мощности точно не известен, как сказал кун выше, что есть гормезис, но есть и эффект Петко, то есть все индивидуально. Выше это предела какого? начинают накапливаться дефекты в генотипе, с которыми организм уже не в состоянии справится и следующие поколения данного индивида обречены. Также стоит отметить влияние детерминированных эффектов и стохастических эффектов, что очень мешает вести статистику и вообще как либо изучать губительное влияние сабжа. Например говорить о участившимся раке щитовидной железы, живших рядом с Чернобыльской АЭС и облучении нет смысла, так как даже школьнику ясно, что более тяжелый и радиоактивный йод занял место в организме и начал облучать его.

Аноним Срд 09 Июл 2014 17:39:04 #105 №116765 

Куда падают разгонные блоки (типа Бриз, Фрегат итд) российских РН.

Они не выводятся на орбиту захоронения?

Аноним Срд 09 Июл 2014 19:53:50 #106 №116795 

>>116765
>Куда падают разгонные блоки
Вниз.
Сгорают в атмосфере. Главное, что бы баки были не из титана, а то их находят всякие пидорасы и потом во всех желтых газетенках пишут про хренопланетян.
http://paranormal-news.ru/news/metallicheskie_shary_s_neba/2011-10-15-3873

Аноним Срд 09 Июл 2014 19:55:04 #107 №116797 

>>116691
Просто пересказываю то, что слушал на курсе радиационной биофизики. Дело не вчера было, могу и ошибаться. Но где у меня там в посте охуительные переведенные открытия? Пруфы чего, гормезиса? Ебать мой хуй, у меня диплом был по гормезису (правда не по радиационному, а по ПОЛ), это давно всем очевидная вещь.
>>116765
Резко и сразу, без проверки предположу, что разгонные блоки оказываются на той же орбите (с небольшими хуями), что и полезная нагрузка - выглядит очевидным.
А зачем их на орбиту захоронения, там же не РИТЕГи стоят.

Аноним Срд 09 Июл 2014 20:25:03 #108 №116804 

>>116795
>Вниз. Сгорают в атмосфере. Главное, что бы баки были не из титана, а то их находят всякие пидорасы и потом во всех желтых газетенках пишут про хренопланетян.

Это понятно. Но я больше беспокоюсь не про уфологов, а про экологов.

>А зачем их на орбиту захоронения, там же не РИТЕГи стоят.
Там баки с гептилом.

Вот сейчас запустили "экологически чистую" (керосин, а могли бы кислород-водород) ракету. Ну, ступени это понятно. А вот если на землю падает разгонный блок, с гептилом - это уже не так красиво.

Аноним Срд 09 Июл 2014 20:35:34 #109 №116809 

>>116804
>А вот если на землю падает разгонный блок, с гептилом - это уже не так красиво.

Баки к тому времени пустые, а температура самовоспламенения гептила около 250 градусов. Так что на землю баки падают уже пустые, или выгоревшие.

Аноним Срд 09 Июл 2014 20:38:28 #110 №116811 

>>116765
Явно не на LEO. Явно не тормозятся им бы нагрузку вывести, лол. Лучше правда нету ни у кого. Там и остаются, надо же будущим госмическим мусорщикам работу дать.

Аноним Срд 09 Июл 2014 20:49:55 #111 №116814 

>>116753
>Я так понимаю человек может жить под этими лучами,
Ты под ними живешь тащемта. Более того, ты их производишь. Твоё тело испускает радиацию, прикинь?
>какого?
150 мГр вроде. Но их статистика такая статистика, что можно на порядок больше дать и ничо не будет, потому что выборка мала и ловят мизера.
http://vspu2014.ipu.ru/proceedings/prcdngs/6546.pdf
Кстати, по щитовидке в тех областях а именно Украина,Белорусь и Молдова, и так был всегда йододефицит. На мозг не влияет. если что, да и обнаружен благодаря Чернобылю. Но статистику подправить запросто может.

Аноним Срд 09 Июл 2014 20:52:45 #112 №116815 

Несколько сменим фокус вопроса: что если будет аварийное падение ракеты.

В случае с керосино-кислородной ракетой ущерб будет существенно ниже чем с гептильным Протоном. Это позитив.
Негатив - упадёт и разгонный блок, а он заправлен гептилом...

Вопрос - а почему не делают экологически чистые разгонные блоки?

Аноним Срд 09 Июл 2014 20:54:44 #113 №116816 

>>116804
>Но я больше беспокоюсь не про уфологов, а про экологов.

Ну так игнорируй экологов, гептил для растений удобрение, это животные от него дохнут кто не убежал. Длительного вреда нет, НГМД шибко активная субстанция и быстро разлагается в условиях таежных говен. В баке чистом полированном может долго хранится, но я бы лично не стал. Пока для меня стабильность у НГМД немного выше, чем у перекиси водорода, нафиг-нафиг.

Аноним Срд 09 Июл 2014 20:57:21 #114 №116817 

>>116815
>В случае с керосино-кислородной ракетой ущерб будет существенно ниже чем с гептильным Протоном.
А хер его еще знает. Керосин пленкой по водоемам растекается, гептил быстрее перерабатывает природа. Людям от гептила конечно кирдык, так вы уже, экологи, определитесь - вы за людей или за природу?

Аноним Срд 09 Июл 2014 21:17:06 #115 №116821 

>>116815
> Вопрос - а почему не делают экологически чистые разгонные блоки?
Делают. Центавр, например, на кислороде и водороде работает. Или кислород-керосиновые блоки Д и производные.
>>116817
> Керосин пленкой по водоемам растекается
Там большая часть сгорит просто. Не будет там такого ужаса.

Аноним Срд 09 Июл 2014 21:19:09 #116 №116822 

>>116816>>116817
Я всё же больше о людях печусь, о здоровье.

Кислород-водородная ракета самая экологически чистая?

Аноним Срд 09 Июл 2014 21:25:20 #117 №116824 

>>116821
Собственно - почему не сделают "благорастворение на воздусях" в виде полностью экологически чистой ракеты кислород-водород, с таким же разгонным блоком.

Вроде как ещё при Союзе были двигатели, нашел упоминание про РД-0120 и РД-701.

Аноним Срд 09 Июл 2014 21:43:32 #118 №116827 

>>116066
>>116432
>>116435
>>116479
Ебанный стыд...
Во-первых, Алькубьерре.
Во-вторых, не упасть, а создавать вокруг корабля изнутри (иначе кина не будет).
В-третьих, НАСА искривляет пространство на десятимиллионную часть, контролируя это сверхточными интерферометрами, до самого варп-привода здесь - как до Антарктиды раком.

Аноним Срд 09 Июл 2014 21:52:06 #119 №116828 

>>116827
>НАСА искривляет пространство на десятимиллионную часть, контролируя это сверхточными интерферометрами, до самого варп-привода здесь - как до Антарктиды раком.
То искривление пространства, которое требуется для двигателя Алькубьерре, требует негативной энергии, которая не открыта толком даже на бумаге. Так что даже какие-то там эксперименты НАСА по искривлению пространства - это все совершенно из другой оперы.

Аноним Срд 09 Июл 2014 22:33:46 #120 №116832 

Огураны, я являюсь одним из жрецов росатого. недавно увидел вот эту новость - http://www.rosatom.ru/journalist/news/10dd3d80449a721ab6a2b6e920d36ab1 что там росатый с раскосым пилит? неужели атомолёт? По инсайду знаю, что система охлаждения там ебически хитровыебанная и лёгкая по массе.

Аноним Чтв 10 Июл 2014 00:28:57 #121 №116866 
1404937737312.jpg

>>116815
Причин много, и все в пользу НДМГ. Наивысшая удельная плотность (не нужно делать баки йоба-габаритов), высококипящий (не нужно делать йоба-теплозащиту на баки), да и в разгонном блоке этого НДМГ - плюнуть и растереть, даже если ракета наебнется на старте, парочка-пяток-десяток (ну это я уж и то загнул) тонн растекшегося гептила - это вам не несколько сотен тонн. Минус - его магические свойства и удельный импульс. Ну чем-то всегда приходится жертвовать.
>>116824
Всему свое место, чисто теоретически можно ракету и на карамельном топливе сделать, но зачем?

Аноним Чтв 10 Июл 2014 00:43:02 #122 №116872 

>>116866
>парочка-пяток-десяток
5-7 тонн топлива берет Фрегат, например. Бриз 5 тонн жижы возит. Аджена 3-6 тонн.
Более тяжелые блоки уже летают на водороде и кислороде.

Аноним Чтв 10 Июл 2014 00:50:47 #123 №116875 

>>116832 смотри сюды >>115608

Аноним Чтв 10 Июл 2014 02:42:06 #124 №116893 

>>116827
Без того пика паста не так эпична.

Аноним Чтв 10 Июл 2014 05:13:57 #125 №116900 
1404954837936.jpg

>>116827
Ты ждал!
Ты знал!
Сколько раз ты пролистывал этот тред в ожидании заветного слова. Как тебе было тяжело. Но теперь ты сделал свое дело и можешь спокойно ждать переката. Что бы все повторилось снова.

Аноним Чтв 10 Июл 2014 06:12:32 #126 №116904 

Как думаете, что находится за космосом?

Аноним Чтв 10 Июл 2014 06:35:57 #127 №116905 

>>116904
Твоя мамка.

Аноним Чтв 10 Июл 2014 06:52:30 #128 №116906 

>>116694
> Только на больших дозах, близких к летальным.
То есть стохастические эффекты ты вот так просто послал нахуй?

>>116827
Иешфаг в треде.

Аноним Чтв 10 Июл 2014 07:03:25 #129 №116907 

>>116905
ты что меня троллишь?

Аноним Чтв 10 Июл 2014 17:56:55 #130 №117019 

В какой степени космический мусор может стать препятствием для освоения космоса? В соседнем треде стращают, что мы можем вообще так засрать околоземное пространство, что вообще запрем себя на планетке.

Аноним Чтв 10 Июл 2014 18:22:01 #131 №117022 

>>117019
В большой степени. Ладно еще низкие орбиты, они быстро чистятся, но на высоких постепенно начинает твориться пиздец, особенно в последние годы, после нескольких крупных событий. С другой стороны, активизировалась куча исследований по очистке космоса, может и придумают чего - если убрать наиболее крупные куски, проблем в будущем будет куда меньше.

Аноним Чтв 10 Июл 2014 21:31:34 #132 №117044 

Почему не пытаются изготовить химическое топливо с действительно высоким удельным импульсом - хотя бы в десяток км/с. ?

Аноним Чтв 10 Июл 2014 21:57:32 #133 №117047 

>>117044
Потому что кислород с водородом все равно ничто не переплюнет, насколько я помню.

Аноним Чтв 10 Июл 2014 22:07:49 #134 №117052 

>>117047
Кроме разве что втора %%:3 твоя память изменяет тебе с интернетом %%

Аноним Чтв 10 Июл 2014 22:18:00 #135 №117054 

>>117044
Почему не пытаются построить гоночный автомобиль мощностью в тысячу лошадиных сил, движимый спрятанными внутри лошадями?

Аноним Чтв 10 Июл 2014 23:42:54 #136 №117061 

>>117054
с лошадями ростом в 1 метр и массой в 1 килограмм каждая? он получился бы слишком громоздким, очевидно жи

Аноним Птн 11 Июл 2014 00:40:58 #137 №117066 

>>117061
Именно. А теперь попробуй с одной лошадью. Химическое топливо имеет свои ограничения (даже йоба-трехкомпонентный двигатель имеет удельный импульс всего в несколько раз выше, чем у пороховой ракеты), их даже двигатель на антиматерии имеет.

Аноним Птн 11 Июл 2014 08:27:05 #138 №117091 
1405052825478.jpg

>>117044
>хотя бы в десяток км/с
Где-то встречал, что ракета для вывода человека на орбиту с таким УИ была бы размером с автомобиль.

Аноним Птн 11 Июл 2014 09:19:47 #139 №117094 

>>117044
если что-то пойдет не так - вся эта энергия выделится разом.
слишком много одноразовых космонавтов надо, однако.

Аноним Птн 11 Июл 2014 10:35:33 #140 №117107 

>>117091
Иначе говоря, топлива нужно было бы массой примерно с сам корабль (двигатель, корпус, груз).

Аноним Птн 11 Июл 2014 10:53:38 #141 №117109 

Здраститя, я к вам с нубо-быдло-вопросом. Где бы почитать про размеры вселенной/космоса? Если космос (ну или как там эти последствия большого взрыва называются) ограничен и висит в ёбанном нихуя, то откуда взялось это нихуя и как оно может быть бесконечным? Откуда известно, что оно бескончное? Почему все объекты стремятся к форме шара, а ёбанное нихуя бескончное и никуда не стремится? (Может на него есть какое-то физическое воздействие "извне", но в таких масштабах не ощутимое?). А если оно конечное, то что за его пределами? И какие есть альтернативы большого взрыва? Мне желательно в изложении для самых маленьких.

sageАноним Птн 11 Июл 2014 11:00:26 #142 №117110 

>>117109
Мало тебя в /быдлятне говном накормили, уебок.

Аноним Птн 11 Июл 2014 11:13:49 #143 №117112 

>>117094
Протоны тоже падают, но это не является непреодолимой помехой для космонавтики.
>>117107
Две трети топлива от общей массы ракеты.
>>117066
Ну так эти ограничения теоретически вполне можно обойти, повысив теплоту сгорания топлива и повысив его плотность.

Аноним Птн 11 Июл 2014 11:27:56 #144 №117114 

>>117109
>А если оно конечное, то что за его пределами?

Нет у него пределов. Представь поверхность шарика. Размеры у неё конечны, а границ в двухмерном понимании нет. То же самое и со Вселенной.

Аноним Птн 11 Июл 2014 11:30:44 #145 №117115 

>>117044

Потому что это невозможно.

Аноним Птн 11 Июл 2014 12:20:23 #146 №117118 

>>116138
Двигатели тех лет второй раз не запускались, это раз, два - топливо чем осаживать будешь ?

Аноним Птн 11 Июл 2014 12:20:52 #147 №117119 

>>117112
Плотность как-то повысить можно (хотя жидкости практически несжимаемы, ну да похуй, раз пошла такая пьянка, будем забивать молотком электроны в протоны), но отношения к удельному импульсу это не имеет. Каждое топливо имеет вполне определенную теплоту сгорания, она определяется химическими связями в веществе и особенностями реации. Выше жопы здесь принципиально не прыгнуть, химические ракетные двигатели имеют возможности для роста УИ на проценты, ну может на десятки процентов, если какой-нибудь ебанат всё-таки запилит трёхкомпонентный.

Аноним Птн 11 Июл 2014 13:20:02 #148 №117126 

>>117112
> Две трети топлива от общей массы ракеты.
Это без учёта гравитации, сопротивления воздуха и импульса от вращения планеты.

Аноним Птн 11 Июл 2014 13:58:45 #149 №117142 

>>117119
Что даст больший импульс - обычный водород или металлический?
В идеале сделать ракету вообще пневматической, с баками из мономолекулярного вещества и заполнять её нейтральным веществом сжатым до умопомрачительного состояния.

Аноним Птн 11 Июл 2014 14:27:20 #150 №117145 

Есть одна двоюродная сис семи лет. С недавних пор стала угорать по космосу. Какое документальное кинцо о космосе посмотреть школьнице? Я уже сбагрил ей свои старые росмэновские книги, но этого явно недостаточно.

Аноним Птн 11 Июл 2014 14:33:09 #151 №117147 

>>117145
>Какое документальное кинцо о космосе посмотреть школьнице?

Cosmos: A Spacetime Odyssey

Аноним Птн 11 Июл 2014 15:15:48 #152 №117163 

спейсач, как думаешь - сверхпроводники смогут помочь защититься от солнечной радиации при полете к Марсу, например.
суть идеи - отклонение сверхмощным магнитным полем частиц с высокой энергией.
оплетем весь полезный объем снаружи по периметру и будем пропускать ток, например.
взлетит?

Аноним Птн 11 Июл 2014 15:26:39 #153 №117164 

>>117163
Магнитное поле надо довольно большое создавать, да и радиация там не такая уж высокая, как пугают. Но в принципе исследуется это дело, я уж не знаю как там со сверхпроводниками.

Аноним Птн 11 Июл 2014 15:41:26 #154 №117165 

>>117164
ну так сверхпроводники его и создают немалое.
а если разработают новые материалы, может еще больше смогут.
опять же.
в педивикии написано, что практически 100 % - это положительно заряженные частицы.
заряжаем поверхность положительно до определенной степени - само все оттолкнется. так в чем проблема то?

Аноним Суб 12 Июл 2014 01:37:46 #155 №117209 

Давеча по телявизору показали, что какое-то суперлуние надвигается. Вопрос: как онлайн без регистрации и смс определить траекторию луны на небе с моего местоположения? Что-то гугление не помогает.

Аноним Суб 12 Июл 2014 02:47:24 #156 №117212 
1405118844690.jpg

Мне вот интересно, если мы не единственные умные существа в нашей галактике, то почему мы до сих пор не уловили радиосигналы инопланетных цивилизаций? Как то грустно осознавать, что мы можем быть одни. да я знаю, что сигнал может пиздецки долго идти с другого конца галактики, но всё же

Аноним Суб 12 Июл 2014 03:26:20 #157 №117213 

>>117212
Умные существа умные и не шкварятся об контакты с землянами

Аноним Суб 12 Июл 2014 07:08:06 #158 №117219 

Как такое может быть, что вселенная бесконечна? Где её начало и где конец? И что за космосом? Как такое может быть, что там нет ничего, лол. Как так что просто так начался взрыв блять из ниоткуда мне кто-нибудь объяснит?

Аноним Суб 12 Июл 2014 08:02:58 #159 №117222 

>>117209
Планетариев довольно дохуя. StarCalc, Stellarium, Cartes du Ciel, тысячи их.

Аноним Суб 12 Июл 2014 08:32:18 #160 №117223 

>>117212
Ну а прикинь, о чем бы можно было поболтать с инопланяшами:

-Привет :3
Через 10000 лет:
-Превед, кагдила?
Еще через 10000:
-Нормик)))000)))
Через 10000:
-Ясн)

Аноним Суб 12 Июл 2014 08:35:30 #161 №117224 

>>117219
Не было никакого взрыва и космоса никакого не было

Было только БОЛЬШОЕ РАЗГИБАНИЕ просранства. Впрочем оно и сейчас есть, только мееедленное

Аноним Суб 12 Июл 2014 17:31:33 #162 №117255 

>>117223 Не в этом дело, чтобы болтать, а дело в самом радиосигнала как доказательство того, что мы не одни.

Аноним Суб 12 Июл 2014 17:40:59 #163 №117258 

Дворщ, посоветую хороших документалок про космос этого года.

Аноним Суб 12 Июл 2014 18:19:08 #164 №117266 

>>117258
Тип, "Новости космоса: Итоги года"?

Аноним Суб 12 Июл 2014 18:24:22 #165 №117267 

>>117266
Не, типа как было "как устроена вселенная серия", но только не такая тупая.

Аноним Суб 12 Июл 2014 18:53:49 #166 №117271 

>>117255
Иноплонетяны же те дураки, чтоб использовать такое тормозное средство передачи данных, какой в жопу радиосигнал? Ты как индеец, недоумевающий, какого хера современные люди не отвечают ему, сигналя дымом.

Аноним Суб 12 Июл 2014 18:57:00 #167 №117273 

>>117271
А что надо?
ПОДПРОСТРАНСТВЕННЫЙ СИГНАЛ?

Аноним Суб 12 Июл 2014 19:49:17 #168 №117277 

>>117273
В 18 веке люди тоже не знали, что есть во вселенной всякое такое "радио". Или ты думаешь, что у нас уже ТЕХНОСИНГУЛЯРНОСТЬ и всё уже открыто?

Аноним Суб 12 Июл 2014 19:52:17 #169 №117278 

>>117277
Так я и спрашиваю, что в теории?

Аноним Суб 12 Июл 2014 20:54:00 #170 №117283 

>>117278
Я чёт вопроса не понимаю. Что "что в теории"? Ояебу, чё в теории? Думаю, в движке вселенной полно всякой хуйни, о которой мы не подозреваем, и жаловаться, что мы не слышим радиосигнал, который мы решили, что надо использовать для передачи инфы, глупо - можа, пришлецы используют особые пердильные железы и общаются, телепортируя обьёмы пердежа друг другу под нос.

Аноним Суб 12 Июл 2014 23:21:23 #171 №117299 

>>117212
Ну смотри. Вселенной 13,8 миллиарда лет. О каком-то маломальском приеме сигналов из космоса человечество может думать последних лет 40-50. Радиосигнал ослабевает и смешивается с фоном на расстояниях порядка нескольких световых лет, насколько я знаю. Уловить и расшифровать его можно, но только если специально искать. Таким образом, даже если откуда-то с ближайших звезд нам в последние 20 лет что-то приходило, то мы это проебали. И проебём скорей всего. Тем временем, иноплане-тяны должны ухитриться существовать с нами практически в одно время, с поправкой на расстояние. Не тысячу лет назад, и уж, тем более, не миллиард. Даже не сто! Таким образом, какая-либо вероятность на мой взгляд, настолько мала, что скорее метеорит на мою лысую голову упадёт и отскочит.

Допустим, во Вселенной жизнь окажется делом рядовым, даже в Солнечной системе мы в итоге найдем каких-нибудь инопланетных простейших и будем ожидать их наличия практически у каждой третьей звезды. Допустим прямо сейчас существует тысяча разумных цивилизаций уровня человечества. Но даже эта тысяча будет так рассеяна, что до ближейшей могут быть сотни лет. Даже так мы хуй что от них получим, слишком мало и слишком поверхностно мы сканируем космос, слишком мало времени прошло. Короче, на радиосигнал я бы не рассчитывал. Я бы ждал или искал какие-нибудь дешёвые и массово произведенные зонды размером с чайник Расселафутбольный мяч, разосланные неведомыми, но могущественными инопланетянками миллионы лет назад по всем звёздам.

Аноним Вск 13 Июл 2014 00:19:25 #172 №117309 

>>117299
> Я бы ждал или искал
Я бы ждал или искал нечто навроде штамма "андромеда", а зонды демаскируют хоумворлд.

и вообще, у мну "фиаско" от такой постановки вброса

Аноним Вск 13 Июл 2014 00:53:31 #173 №117321 

>>117309
Нихуя не понял твоего ответа.

Аноним Вск 13 Июл 2014 01:04:46 #174 №117326 

>>117321
http://lmgtfy.com/?q=%D1%88%D1%82%D0%B0%D0%BC%D0%BC+%D0%B0%D0%BD%D0%B4%D1%80%D0%BE%D0%BC%D0%B5%D0%B4%D0%B0

http://lmgtfy.com/?q=%D1%84%D0%B8%D0%B0%D1%81%D0%BA%D0%BE+%D0%BB%D0%B5%D0%BC

Аноним Вск 13 Июл 2014 01:17:58 #175 №117331 

>>117326
Зело ты мудаковатая личность, потратившая 10 минут на создание этих линков (особенно второй охуенен), вместо того, чтобы вбросить пару строк пояснений к своему предыдущему высеру, написанному так, словно его автор принял на грудь поллитра водки.

Думаю, смысла дискутировать дальше нет.

Аноним Вск 13 Июл 2014 01:49:04 #176 №117332 

>>117326
Извини, вспылил.

Аноним Вск 13 Июл 2014 02:42:03 #177 №117335 
1405204923402.png

>>117334

Аноним Вск 13 Июл 2014 05:31:23 #178 №117348 
1405215083940.jpg

Пока спейсач тралится мамиными тральщиками, вброшу такой, давно интересующий меня вопрос.

Реально ли выйти на орбиту Деймоса? У вторая космическая в несколько метров в секунду, а значит, бросив камень, мы можем ожидать, что вскоре он прилетит нам в затылок. Или Марс делает это невозможным и камень окажется на орбите наблюдения и вскоре лет через тысячу ебнется обратно на эту десятикилометровую скалу? Вообще, века столкновений с микрометеоритами должны превращать такие безатмосферные тела в матку с роящимися вокруг неё астероидами размерами от пылинки до куриного яйца. Не так?

Аноним Вск 13 Июл 2014 05:35:00 #179 №117349 

>>117348
Пости своё ониме в другом месте.
Не тот тралль

Аноним Вск 13 Июл 2014 05:47:21 #180 №117351 
1405216041193.jpg

>>117349
Пости свои сообщения на другом сайте.

Аноним Вск 13 Июл 2014 05:54:56 #181 №117353 

Ладно, хуй с вами. Зарепортил всё даунство, подожду, пока тралли перебесятся или их побанят.
съебал с доски

Аноним Вск 13 Июл 2014 07:36:42 #182 №117367 
1405222602823.jpg

>>117348
> Реально ли выйти на орбиту Деймоса?
Стабильные орбиты у него есть, но очень неравномерные и их расчет, как и в случае любых подобных тел - задача нетривиальная, т.к. есть несколько затыков:

1. Гравитация очень маленькая, поэтому неудачное шевеление рукой или манипулятором или выбрасывание гарпуна может сильно поменять орбиту. АМС, прибывшей к Деймосу, придётся очень точно рассчитывать моменты.

2. Деймос находится дальше от предела Роша чем Фобос, но приливные силы все равно довольно сильно влияют на общую картину, и притяжение с "марсовой" стороны меньше, чем с дальней. (и с марсовой стороны камень будет тянуть чуть сильнее в сторону ретрогрейда)

3. Деймос далек по форме от шара, поэтому гравитационная модель нуждается в вычислительно сложном моделировании (polyhedral gravity model).

Можно в принципе потыкать ODTBX и найти стабильную орбиту для камня. Но с любом случае АМС у Деймоса скорее всего будет полагаться не не только на орбиту, а на автоматику и двигатели малой тяги, для надежности.

> Вообще, века столкновений с микрометеоритами должны превращать такие безатмосферные тела в матку с роящимися вокруг неё астероидами размерами от пылинки до куриного яйца. Не так?
В целом так. Но скорее пылинки, и не слишком много.

Аноним Вск 13 Июл 2014 07:38:43 #183 №117368 

>>117367
> в сторону прогрейда
фикс

Аноним Вск 13 Июл 2014 22:40:48 #184 №117453 
1405276848746.jpg

интересная статья с хабры.
необразованным идиот весьма будет интересно и познавательно почитать. рекомендую.

Не лей мне соль в реактор или не-импульсные ядерные ракетные двигатели
http://habrahabr.ru/post/227973/

Аноним Вск 13 Июл 2014 23:16:57 #185 №117457 

>>117453
Жаль только там ничего не говорится о том, что нормальный ЯРД не так-то просто получить, как об этом думается в теории.

Аноним Вск 13 Июл 2014 23:30:16 #186 №117458 

>>117453
> необразованным идиот
т.е. ты как бы хочешь нам сказать, что статья полна фактических ошибок и вообще танкий траленк?

Аноним Вск 13 Июл 2014 23:52:09 #187 №117460 
1405281129900.png

>>117367
Копался на эту тему и нашел довольно любопытный обзор парней из JPL http://trs-new.jpl.nasa.gov/dspace/bitstream/2014/42478/1/12-3014.pdf, в котором все расписано довольно подробно. Там в основном про Фобос, но про Деймос почти все будет аналогично. Презенташка к работе: http://trs-new.jpl.nasa.gov/dspace/bitstream/2014/43035/1/12-3712_A1b.pdf

Бля, вот под такие вопросы надо отдельные треды пилить.

Аноним Пнд 14 Июл 2014 05:56:16 #188 №117484 
1405302976320.png

>>117367>>117460
Спасибо. Кое-как осилил пейперы. Матан с графиками мне малопонятны, кроме того, что видно, что обозначенные параметры меняются со временем, но по орбитам вполне ясно. Классическая круговая орбита невозможна ввиду специфики Фобоса и Деймоса, но возможна DRO (как по-русски?). Как я понимаю, в этом случае орбиты КА и Фобоса с Деймосом просто пересекаются, как олимпийские кольца, но основным "гравитатором" является Марс, при этом КА будет делать витки вокруг спутника, причем, он может делать это довольно близко к поверхности. Также возможны хитрые лагранжевые орбиты - Ляпунова, вертикальная и гало, представляющие собой витки (орбита Ляпунова, гало-орбита) и восьмерки (вертикальная) вокруг L1 и L2. Вертикальная, кстати, на русском тоже вроде орбитой Ляпунова называется, только вертикальной. Принцип висения на лагранжевых орбитах стал понятен после прочтения раздела про орбиты Ляпунова тут: http://sinsam.kirsoft.com.ru/KSNews_260.htm
Там же чуть ниже нарисован интересный и дешёвый трансфер из L2 в L1. Сами по себе эти орбиты нестабильны и аппарат норовит с них сойти, но находиться на них можно относительно "дёшево", понемногу включая двигатели.

Аноним Пнд 14 Июл 2014 08:15:55 #189 №117489 

>>117163
Магнитное поле только на заряженные частиц подействует. А это либо электроны, либо ионы. От вторых защититься легко. От первых тоже можно защититься, но отклонять их, это неплохой вариант.
А от гамма-излучения магнитным полем не защитишься. А его в космосе тоже предостаточно.

>>117165
> заряжаем поверхность положительно до определенной степени - само все оттолкнется. так в чем проблема то?
У тех заряженных части такая энергия, что они спокойно преодолевают поле от заряда ядра и даже могут попасть в это ядро, и даже хватит энергии на какой-нибудь микроскопический бадабум. А ты предлагаешь макроскопическим зарядом их отклонять.

>>117255
Мы начали срать радиосигналами примерно 100 лет назад. Следовательно, наши радиосигналы ушли от нас максимум на 100 световых лет. Аналогичная ситуация с инопланетянами. Если они на другом конце галактики и появились относительно недавно, то мы ещё долго не поймаем их радиосигналы.

Аноним Пнд 14 Июл 2014 11:41:20 #190 №117497 

Как вы думаете, есть ли что-нибудь за космосом или нет?

Аноним Пнд 14 Июл 2014 11:48:14 #191 №117499 
1405324094708.jpg

>>117497

Аноним Пнд 14 Июл 2014 12:10:53 #192 №117501 

>>117499
что ты сразу на личности переходишь? ты просто попробуй представить это, что забкосмосом что-то есть. сразу крыша съезжает? вот и я о том же

Аноним Пнд 14 Июл 2014 12:20:51 #193 №117505 

>>117457
Схуяли? Что наш, что американский ЯРД работали вполне нормально, никто не жаловался.
мимокрокодил

Аноним Пнд 14 Июл 2014 12:34:57 #194 №117509 

>>117505
Проблему утери активного вещества так и не удалось решить. Газофазник не взлетел. Твердотельный получился на редкость говенным, слабым, тяжелым и низкоресурсным. Солевые и импульсные вообще пиздец. Один полет и потом десятки лет магнитосферу Земли вычищать от изотопов.

Аноним Пнд 14 Июл 2014 21:15:34 #195 №117592 
1405358134918.jpg

Кто-нибудь знает что это по орбите ISS прямо за ней летит?

Аноним Пнд 14 Июл 2014 21:22:33 #196 №117596 

>>117592
Cygnus запущенный украинской ракетой antares.

Аноним Пнд 14 Июл 2014 21:45:15 #197 №117602 

>>117299
>метеорит на мою лысую голову упадёт и отскочит
Batya, это ты?

Аноним Втр 15 Июл 2014 01:06:35 #198 №117619 

>>115935
Анон поясни.
1. Что в реальной ракете и/или космическом корабле заменяет KSPшный модуль SAS?
2. Как он устроен? Где он находится? Может ли ракета или космический корабль поворачивать только с помощью гироскопа как в KSP?
3. Каким образом изменяется вектор тяги двигателя? Как устроен механизм регулирования?

Аноним Втр 15 Июл 2014 01:51:30 #199 №117620 
1405374690770.jpg

>>117619
1. Такое же структурное кольцо, на котором монтированы датчики и эвм для контроля за движением/параметрами и прочие мозги ракеты.
2. Находится повыше, на 2 ступени обычно т.к. одно на всю ракету. Пикрелейтед сверху - сатурновская SAS, здоровенная ёба. Космический кораблю/станция/спутник может и применяется. Называется этот девайс гиродин - сжирает вредное вращение за счёт передачи на свои маховики. Обязательны на спутниках и мкс (здоровенные) для экономии топлива и стабилизации.
3. Камера сгорания вместе с соплом и турбокомпрессором (обычно) подвешены на карданов подвес (gimbal ring), за приводы (actuator) которого можно поворачивать двигло целиком в разные стороны (на пике РД-171). На ранних ракетах поворот за счёт маленьких управляющих сопел - РД-108 Востока на пике. На Фау-2 - рули из карбидного композита в реактивной струе.

Аноним Втр 15 Июл 2014 02:05:35 #200 №117622 

http://www.gearthblog.com/satellites

Пиздец блдяь, все засрали своими спутниками, уебки.

Аноним Втр 15 Июл 2014 02:17:57 #201 №117625 

>>117619
Это совсем не тупой вопрос, кстати.

1. Стабилизация по крену при взлете ракеты обычно делается газовыми рулями ориентации (по типу RCS в огурцах). Стабилизация по остальным двум осям - либо так же, либо вращением (чаще бывает в геофизических ракетах), либо изменением вектора тяги (в орбитальных РН), либо дифференциальной тягой, если у ракеты несколько двигателей.

2. Может, и поворачивает. Разница в том, что в реальных гиродинах чтобы повернуть - нужно ускорить или замедлить колеса. Бесконечно ускоряться они не могут - их распидорасит. Кроме того, быстро вращающаяся хуйня оказывает стабилизирующий эффект, который может быть нежелателен. Поэтому постоянно противодействовать внешней силе, как в огурцах, не получится, и их не применяют когда на корабль действуют большие внешние силы (например в атмосфере). Тем не менее, солнечный ветер/давление света/магнитные силы etc. придают небольшой крутящий момент, и приходится крутить их всё быстрее, поэтому время от времени надо делать их разгрузку, дуя двигателями ориентации в обратную сторону и замедляя вращение. Соответственно, без RCS для длительно существующих спутников/станций все равно не обойтись. В видеорилейтеде разгрузка достигается за счет опоры, в космосе опереться не на что, кроме струи двигателя. Кроме того, настоящие гиродины придают очень медленное вращение кораблю, крутиться как волчок туда-сюда не получится (да и не надо). Ещё у них есть такая особенность как "сингулярности": есть некоторые позиции в пространстве, в которых как бы ты ни крутил гироскоп, он тебе не даст противодействующего момента в нужную сторону.

3. Поворотом двигателя на шарнире, собственно.

Аноним Втр 15 Июл 2014 02:43:18 #202 №117628 

Сколько выведет на низкую круговую орбиту Сатурн-5, если его запустить с Марса? А Союз?

Аноним Втр 15 Июл 2014 11:17:20 #203 №117633 

>>117620
Ну не только на ранних ракетах, на шаттловских двигателях тоже использовались десятки малых ЖРД (эти же двигатели, к слову, американцы собираются и теперь использовать - УИ у них весьма йобистый, как я понимаю).
>>117628
Ну смотри. Считаем дельту до низкой опорной для Марса (гравитация + атмосфера + горизонтальная скорость), вычитаем её из дельты для Земли, смотрим какой остается масса ракеты к тому времени, когда исчерпано такое количество дельты, это и будет ПН. Но конечно это тот еще долбоебизм, банально двигатели для Марса надо другие ставить для оптимального выведения - там ведь чуть ли не вакуум на поверхности (если не из Эллады запускаем, а уж если с Олимпа - то совсем вакуум). Если кратко - то с Марса ракету запустить в разы легче чем с Земли.

Аноним Срд 16 Июл 2014 01:02:16 #204 №117767 

>>117633
А чем различаются двигатели для вакуума и для атмосферы?

Аноним Срд 16 Июл 2014 01:16:58 #205 №117769 
1405459018629.jpg

>>117767
Формой сопла, если не вдаваться в незначительные подробности. Разными конфигурациями сопла Лаваля (традиционного, которое стоит на 99% ракетных движков) можно добиться наибольшей тяги в вакууме или атмосфере.

Есть правда сопловые насадки и всякие клиновидные сопла, которые могут работать везде одинаково.

Аноним Срд 16 Июл 2014 01:24:00 #206 №117771 

>>117769
Аэроспайки пока не работают нигде, кроме влажных фантазий одноступенщиков ну и огневых испытаний, таки да. Но подождем - увидим, вон недавно ракету обещали с клиновидным.

Аноним Срд 16 Июл 2014 10:25:54 #207 №117795 

Почему космос черный? Вот на Земле небо. Оно голубое, и эту голубизну создает 15-20 км атмосферы. А в космосе пространства млрд световых лет и космос черный. Он действительно настолько пустой?

Аноним Срд 16 Июл 2014 11:06:19 #208 №117798 
1405494379709.jpg

>>117795
Да.

Аноним Срд 16 Июл 2014 11:08:00 #209 №117799 

>>117795
Не всё так просто, но зато интересно.
http://ru.wikipedia.org/wiki/Фотометрический_парадокс

Аноним Срд 16 Июл 2014 11:08:34 #210 №117800 

> огневых испытаний
Кстати как испытывают двигатели в вакууме? Они же генерируют дикое кол-во газа и вакуум пропадает.

Аноним Срд 16 Июл 2014 13:55:25 #211 №117835 

А действует ли на сам свет релятивистское замедление времени?

Аноним Срд 16 Июл 2014 14:26:23 #212 №117842 

>>117835
У света весьма интересные эффекты получаются в этом плане. Например он всегда движется равномерно и прямолинейно, "исказить" траекторию может только искривление самого пространства.

Аноним Срд 16 Июл 2014 20:59:20 #213 №117875 
1405529960534.jpg

>>117842
Я люпа, сасы мой залюпа!

Аноним Чтв 17 Июл 2014 15:23:36 #214 №117952 
1405596216814.jpg

>>117875
Тебе просто необходимо прочитать пикрелейтед.
Тебе станет понятно, почему между обоими вашими заявлениями нет противоречий.

Аноним Птн 18 Июл 2014 16:26:15 #215 №118256 

Мне грустно от того, что человечество никогда не сможет в межзвездных перелетах. При идеальных условиях до ближайшей звезды лететь ~ 50лет. Если окажется, что кротовых нор нема, или мы не сможем их использовать, то все - пизда, спейсаны.

Аноним Птн 18 Июл 2014 17:12:18 #216 №118270 

>>118256
>При идеальных условиях до ближайшей звезды лететь ~ 50лет.
Лолнет. До Альфы Центавра 2.26 года с ускорением в 1g, до Альфы Треугольника 2.33 года, даже до Туманности Андромеды 30 лет, а это вообще другая галактика. А если человек сможет длительно переносить 2g, то за 11 лет можно пролететь всю вселенную поперек.
http://www.allkosmos.ru/vozmozhny-li-polety-cheloveka-k-drugim-zvezdam-i-drugim-galaktikam/

Аноним Птн 18 Июл 2014 17:39:34 #217 №118277 

>>118270
Формулу в студию, умник, да с пояснениями. Та таблица на сайте - полная хуйня. Ты разгоняешься с ускорением 10м/с (до Центавры 4,36 св лет), а потом тормозишь с тем же ускорением.
Какие 2 года, ебана в рот?

Аноним Птн 18 Июл 2014 17:40:44 #218 №118278 

>>118256
Ты сначала с ближайшим окружением разберись. На наш век космонавтики хватит.

sageАноним Птн 18 Июл 2014 17:41:00 #219 №118279 

>>118277
>этот кэфирный дебилоид уровня обо/сцай

Аноним Птн 18 Июл 2014 17:48:39 #220 №118281 

>>118270
Пиздец ты ебанутый! Расстояние 4,3 св года, а мы, блять, долетим за 2,3 с ускорением 1ЖЕ!
V = 2c
Genius.jpg

Аноним Птн 18 Июл 2014 17:51:15 #221 №118282 

>>118279
Схуяли я кефирный,это этот >>118270 даун утверждает что он превысит скорость света.

Аноним Птн 18 Июл 2014 17:55:54 #222 №118283 

>>118270
До Андромеды 2538000 св лет? Каким образом ты за 30 долетишь, поехавший?

Аноним Птн 18 Июл 2014 18:32:36 #223 №118293 

Все, пиздец. Мой спейсач сдох нахуй, одни дегенераты остались - >>118281 >>118282 >>118283
Эти животные даже не, хаха, поняли что речь идет о локальном релятивистском времени космонавта.
Убить всех, эта попытка не удалась.

Аноним Птн 18 Июл 2014 18:55:44 #224 №118295 

>>118293
> одни дегенераты
Почему одни? Их множество!

Аноним Птн 18 Июл 2014 19:00:10 #225 №118297 

>>118256
не очкуй. Как только учёные изобретут так сразу и полетим . Ну может через пару лет. Надо всего лишь разъебать дядюшку энштейна. Где-то он ошибся

Аноним Птн 18 Июл 2014 19:08:15 #226 №118301 

>>118270
>До Альфы Центавра 2.26 года

Толсто.

Аноним Птн 18 Июл 2014 19:16:38 #227 №118303 

>>118301
Ох лол, а этот не только дегенерат, но еще и слоупок - >>118293

Аноним Птн 18 Июл 2014 19:29:39 #228 №118309 

>>118303
Иди поешь говна, локальное время космонавта не будет меньше времени, которое затратит свет на преодоление расстояния.

Аноним Птн 18 Июл 2014 19:40:21 #229 №118312 

>>118309
Чайку?
>>118293
А ты идешь нахуй.

Аноним Птн 18 Июл 2014 19:41:21 #230 №118313 

>>118293
>речь идет о локальном релятивистском времени космонавта.
Поясни.

Аноним Птн 18 Июл 2014 19:43:06 #231 №118314 

>>118309
>локальное время космонавта не будет меньше времени
А, так ты кефирщик. Или возможно имбецил в медицинском смысле.
>>118312
А вот с этим все ясно полностью, кефир обычный бесполезный.

Аноним Птн 18 Июл 2014 19:44:38 #232 №118315 
1405698278273.gif

>>118313
Попробуй посчитать.
Если, конечно, есть чем.

Аноним Птн 18 Июл 2014 19:52:37 #233 №118319 

>>118314
Так, всем спокойно! Ясно, что при скоростях близким к скорости света время замедляется, и если летать по орбите черной дыры пару лет, то на Земле пройдет overDOHUYA. Но, блджад, неужели путь к Андромеде (как кто-то писал 2538000 св лет), с точки зрения космонавта займет 30 лет?? Пиздежом попахивает.

Аноним Птн 18 Июл 2014 19:57:43 #234 №118320 

>>118319
Просто ты дебил и не можешь в арифметику. Это ничего, для быдла норм.
А скорость на пути к Андромеде будет где-то 99.999много9% с - сколько девяток точно посчитай сам. И, соответственно, безумное же лоренцево сокращение времени.
Правда, на такой скорости первый же встречный атом нахуй испарит корабль и все окружающее - но это уже другой вопрос.

Аноним Птн 18 Июл 2014 20:04:31 #235 №118322 

>>118320
>встречный атом
Хм. Погорячился, пожалуй, галактические высокоэнергетические немногим слабее. Пылинка - точно испарит, а атомы просто превратят вторичным облучением железяку в летающий чернобыль.

Аноним Птн 18 Июл 2014 20:20:26 #236 №118323 

так если учёные придумают как летать к звёздам это же создаст и другие технологии? Которые разрушат нашу экономику и возможно цивилизацию? Телепорт, антиграв и т.д. Что в первую очередь будет использовано как оружие.
Получается мы никогда не полетим к звёздам?

sageАноним Птн 18 Июл 2014 20:28:00 #237 №118324 

>>118323
>учёные
в говне моченые

Аноним Птн 18 Июл 2014 22:34:49 #238 №118342 
1405708489481.jpg

Что за заплаька на Луне? Явно же указывает на искусственное происхождение, разве не?

Аноним Птн 18 Июл 2014 22:38:28 #239 №118344 

>>118342
Заплатка

Аноним Птн 18 Июл 2014 22:51:53 #240 №118347 
1405709513181.gif

>>118342
> Явно же указывает на искусственное происхождение, разве не?
Искусственное, всё верно.

Аноним Птн 18 Июл 2014 23:07:58 #241 №118352 

>>118342 да по версии google map, таких заплаток и на Земле полно, лол )
Просто разные фото из разных источников. Так спутник пролетел просто.

Аноним Суб 19 Июл 2014 04:01:27 #242 №118379 

>>118342
Там ещё правее поля распаханы полосами, темнее-светлее. Это лунатики разные сельскохозяйственные культуры там сажают.

Аноним Вск 20 Июл 2014 02:06:06 #243 №118548 
1405807566706.jpg

Антон, как перемещаться человеку при большом ускорении, что бы не распидорасило? Есть ли какие идеи, концепции, проекты ИРЛ?

Аноним Вск 20 Июл 2014 02:53:10 #244 №118550 

>>118548
Всё что сложнее индивидуально подогнанных противоперегрузочных кресел и тренировки - влажные фантазии. Из таковых имеются плавание в насыщенном кислородом физрастворе, например. Насчет состояния экспериментов по этой теме я хз, правда. В любом случае порог поднимается не сильно, а геморрой сильно.

Аноним Вск 20 Июл 2014 21:06:28 #245 №118650 

>>118320
>А скорость на пути к Андромеде будет где-то 99.999много9% с

Давай, расскажи-ка нам, про двигатель и топливо, которые будет поддерживать ускорение 1G, при возрастающей массе КА, приближающегося к 99.9% с

Аноним Вск 20 Июл 2014 22:11:59 #246 №118659 

>>118650
>возрастающей массе
Ещё одна бредящая релява, даже не читавшая свою библию.

Аноним Вск 20 Июл 2014 22:45:12 #247 №118662 

>>118650
>про двигатель и топливо
Статью даже не читал, значит. Ну да что с неграмотного дебила взять.
Исходное условие - 1 g на протяжении всего пути, все остальное результат. Разумеется, далее в статье указано что даже к соседним звездам вряд ли возможно, к галактикам - никак и никогда, ибо нехуй.
>>118659
Ну а это даже не дебил - просто животное.
Печаль.

Аноним Вск 20 Июл 2014 23:20:32 #248 №118666 
1405884032939.jpg

Космонач! Где можно посмотреть статистику по количеству спутников у разных стран? Сколько их всего вообще, сколько у какой страны, если возможно, то с разбивкой по категориям: военные, телевизионные, связи, научные и т.д. Интересуют именно данные по нынешнему положению дел, общее количество запущенных за всю историю освоения космоса немного не то.
Гугл чего-то не особо помог.

Аноним Вск 20 Июл 2014 23:51:02 #249 №118667 
1405885862758.jpg

>>118666
Вот так примерно. На пике американская софтина, написанная для оборонки.

Аноним Пнд 21 Июл 2014 01:00:34 #250 №118675 
1405890034419.jpg

>>118667
Инфографика это, конечно, круто, но мне бы в табличном виде. Сколько штук у кого и каких. Цифры, они поинтереснее будут, как мне кажется.

Аноним Пнд 21 Июл 2014 10:17:22 #251 №118705 
1405923442974.png

>>118666
> Где можно посмотреть статистику по количеству спутников у разных стран?
Из первоисточников - организаций, отслеживающих космические объекты.
http://orbitaldebris.jsc.nasa.gov/
Не знаю, доступны ли отчеты ГЦ РКО.
Всё это, понятное дело, только по околоземным объектам - космическому мусору, действующим и мёртвым спутникам. АМС например там не учитываются.

> если возможно, то с разбивкой по категориям: военные, телевизионные, связи, научные и т.д.
Кроме вышеуказанного - попробуй потыкай софт для трекинга спутников (heavensat, nova, orbitron и т д.), и онлайн-трекеры, во многих из них есть более-менее актуальные каталоги.

Аноним Пнд 21 Июл 2014 12:39:24 #252 №118715 

>>118659
Нехуй возразить - съебал нахуй.

Аноним Пнд 21 Июл 2014 13:59:14 #253 №118725 
1405936754363.png

>>118705
>онлайн-трекеры
А по каким ключам гуглить, няша? А то я полный нуб в этом вопросе и 9/10 выдачи у меня занимают ссылки на спутниковые телевизионные каналы и ресиверы с тарелками к ним.

Аноним Пнд 21 Июл 2014 14:13:45 #254 №118726 

>>118725
online satelltie tracking/orbit tracking, очевидно же.
Сервисов масса, n2yo, satflare и т.п., какие из них актуальней/лучше/хуже - не подскажу. На многих есть и списки по категориям.

Аноним Пнд 21 Июл 2014 15:22:12 #255 №118731 

Можно ли в ближайшие 30 лет построить тороидальную косм. станцию с ценой меньше 5 млн долларов?

88000800 Аноним Пнд 21 Июл 2014 15:45:52 #256 №118737 

>>118731
да конечно
давай мне задаток. Через 5 лет ключи передам

Аноним Пнд 21 Июл 2014 16:13:20 #257 №118741 

Почему на снимках из космоса абсолютно чёрное "небо"? На земляшке я ведь могу увидеть кучу звёзд

Аноним Пнд 21 Июл 2014 17:12:06 #258 №118757 
1405948326865.jpg

>>118731
Хоть через несколько месяцев, за несколько десятков тыщ. Вопрос в размерах.

Аноним Пнд 21 Июл 2014 17:14:30 #259 №118759 

>>118741
Ограниченный динамический диапазон камер.

Аноним Пнд 21 Июл 2014 20:02:44 #260 №118771 
1405958564756.jpg

>>118741
Но сфогграфирать - не можеж.

А он, приклейтед - может. правда, он не может их увидеть, но это совсем другая история

Аноним Пнд 21 Июл 2014 20:11:19 #261 №118775 

>>118731
> Можно ли
Да, можно, я разрешаю.

Можно и избуханки сделать троллейбус, но зачем.

Аноним Втр 22 Июл 2014 01:51:31 #262 №118871 

Почему Венера вращается в противоположную сторону, в отличие от остальных планет?

Аноним Втр 22 Июл 2014 02:27:32 #263 №118873 
1405981652755.jpg

Где годные помоечки с хайрезными фотками МКС, КА и прочей космонавтики? Чтобы поменьше рож и побольше пикрелейтедов.

Аноним Втр 22 Июл 2014 02:31:54 #264 №118874 

Почему нельзя преодолеть скорость света?

Аноним Втр 22 Июл 2014 03:01:10 #265 №118877 

>>118874
http://ru.wikipedia.org/wiki/%D1%EA%EE%F0%EE%F1%F2%FC_%F1%E2%E5%F2%E0#.D0.92.D0.B5.D1.80.D1.85.D0.BD.D0.B8.D0.B9_.D0.BF.D1.80.D0.B5.D0.B4.D0.B5.D0.BB_.D1.81.D0.BA.D0.BE.D1.80.D0.BE.D1.81.D1.82.D0.B8

>>118871
Ещё Уран вращается в обратную сторону, правда он почти "лежит на боку".

Аноним Втр 22 Июл 2014 03:16:17 #266 №118878 

>>118877
А если без теории относительности?

Аноним Втр 22 Июл 2014 04:50:46 #267 №118883 

>>118877
>лежит на боку
или
>вращается в обратную сторону
Так почему?

Аноним Втр 22 Июл 2014 04:53:34 #268 №118884 

>>118883
Вероятно, въебалось что-то очень крупное на заре времён, так и povernoolo.

Аноним Втр 22 Июл 2014 05:00:08 #269 №118885 

>>118878
Информация любое взаимодействие распространяется не быстрее скорости света, иначе попадет в прошлое.

Аноним Втр 22 Июл 2014 06:26:08 #270 №118887 

>>118883
Пробные запуски планетарного двигателя.
http://youtu.be/F4NlX_MOe6I?t=1m49s

Аноним Втр 22 Июл 2014 12:35:38 #271 №118911 

>>118871
Точно не известно.
Есть несколько гипотез.
Например, из-за того, что в нее что-то въебалось, еще есть теория, что такое возможно из-за поведения атмосферы Венеры.
Так же есть гипотеза о внешнем влиянии притяжений Солнца, Земли и Юпитера.
Но окончательного ответа не существует.

Аноним Втр 22 Июл 2014 12:37:16 #272 №118912 

>>118885
прошлое для наблюдателя который об этом узнает только спустя время
в этом нет ничего необычного

Аноним Втр 22 Июл 2014 12:39:10 #273 №118913 

Анон, что эффективнее и надёжнее? Солнечные батареи или "mercurium boils" есть вогнутое зеркало фокусирующее свет на трубке внутри которой течёт ртуть которая крутит лопасти турбины ?

Аноним Втр 22 Июл 2014 12:41:44 #274 №118914 

Олсо, взлетит
http://science.compulenta.ru/684842/
?

Аноним Втр 22 Июл 2014 13:14:57 #275 №118916 

>>118885
Ты меня еще больше запутал. Вот удалось разогнать ракету до сверхсветовой. Как она попадет в прошлое?

Аноним Втр 22 Июл 2014 15:05:22 #276 №118925 

>>118878
>А если без теории относительности?
То придумай себе нужную теорию и объясни себе следствия из неё. Ты бы ещё попросил объяснить, почему голубиная какаха тебе на голову падает, только без теории гравитации.

Аноним Втр 22 Июл 2014 15:46:03 #277 №118926 

>>118916
Заебал, есть куча книжек по физике с элементарными задачками, которые тебе объяснять всю суть мироздания.

Аноним Втр 22 Июл 2014 19:40:59 #278 №118950 

Почему вы считаете что Бога нет?

Аноним Втр 22 Июл 2014 19:48:59 #279 №118956 

>>118950
define Б-г

Я не считаю, что его совсем нет. Однако, его существование уже не является необходимым для абиснения подавляющего большинства фактов.
Ну и оставшиеся факты скоро объяснят без привлечения, скорее всего.

Короче. Б-г нинужен.

Аноним Втр 22 Июл 2014 19:53:13 #280 №118957 

>>118874
Можно. Преодолевай, превозмогай, я разрешаю. Если кто спросит - скажешь - я разрешил.

ее нельзя не то что преодолеть, но даже приблизиться. Никакой мочи не хватит. Масса мешает

Аноним Втр 22 Июл 2014 19:55:29 #281 №118958 
1406044529443.jpg

Аноны, а кто знает, есть ли теоретический способ попасть в прошлое? Возможно ли вообще такое по сути то своей?

Аноним Втр 22 Июл 2014 20:08:41 #282 №118962 

>>118950
А почему я должен считать, что он есть?

Аноним Втр 22 Июл 2014 20:12:50 #283 №118963 

>>118958
>есть ли теоретический способ попасть в прошлое?
По хардкору:
http://www.razlib.ru/fizika/kratchaishaja_istorija_vremeni/p11.php

Аноним Втр 22 Июл 2014 21:50:21 #284 №118979 

>>118963
Допустим, к примеру, что событие А — это финиш заключительного стометрового забега на Олимпийских играх 2012 г ., а событие В — открытие 100004-го Конгресса альфы Центавра. Допустим, что для наблюдателя на Земле событие А предшествует событию В. Скажем, событие В происходит годом позже — в 2013 г . по времени Земли. Так как Земля и альфа Центавра разделены расстоянием около четырех световых лет, эти два события удовлетворяют вышеупомянутому критерию: хотя А случается прежде В, чтобы поспеть от А к В, вы должны перемещаться быстрее света. В таких обстоятельствах наблюдателю на альфе Центавра, удаляющемуся от Земли с околосветовой скоростью, казалось бы, что события имеют обратный порядок: событие В происходит раньше события А. Этот наблюдатель утверждал бы, что, перемещаясь быстрее света, можно поспеть от события В к событию А. Следовательно, обладай вы способностью обгонять свет, смогли бы вернуться обратно от А к В до начала забега и сделать ставку, зная наверняка, кто победит!

Это какой-то ебаный бред. Есть другое объяснение?

Аноним Втр 22 Июл 2014 22:45:43 #285 №118984 

>>118979
По-моему, объяснить ещё проще уже невозможно. Забей, посвяти себя гуманитарным предметам.

Аноним Втр 22 Июл 2014 23:45:00 #286 №118988 

>>118979
>Это какой-то ебаный бред. Есть другое объяснение?
Тебе нужно начать с азов каких-то, если тебе даже это не понятно.
Хотя бы вот этот научпопс глянь что ли.

Аноним Срд 23 Июл 2014 00:16:26 #287 №118994 

>>118988
25-26 минуты.
>Если я вылечу со сверхсветовой, то вернусь и пожму руку себе самому.
Почему, он так и не объясняет.

Аноним Срд 23 Июл 2014 00:40:18 #288 №118999 

Во многих передачах про звезды, когда речь заходит о нейтронных звездах, говорят о том, что чайная ложка вещества с звезды на Земле будет весить кучу тонн. Вопрос: если дотащить эту чайную ложку до Земли, вещество в ней останется такого же размера или же в отсутствии сильной гравитации его раздует до размеров, соответствующих его массе?

Аноним Срд 23 Июл 2014 01:20:26 #289 №119001 

>>118999
В таком состоянии вещество держится только гравитацией, превосходящей слабые силы, так что если "вынуть" кусок из звезды, начнётся распад нейтронов, и этот кусок взорвётся с чудовищной силой.

Аноним Срд 23 Июл 2014 01:22:53 #290 №119002 

>>118999
>раздует до размеров, соответствующих его массе
this

Аноним Срд 23 Июл 2014 14:06:03 #291 №119034 

>>119002
Вообще-то нейтронные звёзды состоят из вырожденного нейтронного газа. Какой объём соответствует некоторой массе газа, а? У него есть только минимальный объём, соответствующий максимальной плотности данного вырожденного газа.
Ну и да, без соответствующей гравитации оно ебанёт так ебанёт.

Аноним Срд 23 Июл 2014 14:15:33 #292 №119035 

>>119001
>если "вынуть" кусок из звезды
На что затратится примерно та же энергия, что выделится при бабахе.
Дополнил, не благодари.

Аноним Срд 23 Июл 2014 15:35:13 #293 №119045 

>>118979
Тебя не смущает, что ты можешь увидеть одновременно две вспышки молнии, а раскаты грома услышать с разницей в несколько секунд?

Аноним Срд 23 Июл 2014 16:14:46 #294 №119051 

>>119035
Это с чего ты взял?

Аноним Срд 23 Июл 2014 16:26:36 #295 №119052 

>>119051
С закона сохранения энергии, очевидно же.
Но кефиропетухам, конечно, закон не писан - так что проходи, не задерживайся, тебя эфир ждет.

Аноним Срд 23 Июл 2014 19:19:43 #296 №119072 

>>119052
Толстовато, кефиропетух, толстовато. Впрочем, к тебе вопросов нет - так что проходи, не задерживайся, тебя эфир ждет.

Аноним Срд 23 Июл 2014 19:27:33 #297 №119074 

>>119045
Нет, не смущает. Все абсолютно логично.

Аноним Срд 23 Июл 2014 19:56:10 #298 №119081 

>>119072
>нет ты сам дурак
Какое самовозгорания, хаха.
А если учитывать вот эти твои посты - >>119074 >>118979 - то ты просто глуп и палишься по причине банальной умственной неполноценности.
Это норма для кефирщиков, впрочем, не огорчайся.

Аноним Срд 23 Июл 2014 20:02:59 #299 №119082 

>>119081
Нет, возгорание здесь только у тебя.
И если учитывать вот эти твои посты - >>118281 >>118874 - то ты просто глуп и палишься по причине банальной умственной неполноценности.
Это норма для кефирщиков, впрочем, не огорчайся.

Аноним Срд 23 Июл 2014 20:08:36 #300 №119085 

>>119081
Чини детектор

Аноним Срд 23 Июл 2014 20:12:10 #301 №119086 
1406131930308.jpg

>>119082
>этот копипаст
Он еще и хохол.
Характерный для йододефицитных уровень дегенерации плюс перманентная истерика в связи с ситуацией на руине - все сходится.

Аноним Срд 23 Июл 2014 20:15:14 #302 №119087 

>>119085
Если предположить хаха что у этих постов разные авторы, то ты или кефиродебил - >>118979 , или самовозгоревшийся простодебил - >>119072
Соболезную в обоих случаях.

Аноним Срд 23 Июл 2014 20:17:15 #303 №119088 
1406132235738.png

>>119087
Че ты несешь, блядь, вот че ты несешь?

Аноним Срд 23 Июл 2014 20:21:41 #304 №119089 
1406132501070.jpg

>>119088
>Че ты несешь
Наношу добро и причиняю справедливость, а что?

Аноним Срд 23 Июл 2014 21:24:57 #305 №119093 

>>119086 На /по/рашу, быдло

Аноним Срд 23 Июл 2014 21:31:45 #306 №119096 
1406136705290.jpg

>>119093
Ясно.

Аноним Срд 23 Июл 2014 22:21:06 #307 №119102 

>>119096
Ладно, парашный дебил, давай попробуем вступить с тобой в беседу, начиная с того поста, в котором ты продемонстрировал свои скудные умственные способности: >>119035
Поясни-ка механику "сохранения энергии", которая уравнивает энергию бета- и гамма-распада некоего количества нейтронов вырожденного нейтронного газа и энергию, необходимую для "извлечения" этого количества газа из звезды.

Аноним Срд 23 Июл 2014 22:40:12 #308 №119105 

>>119102
>этот изобретатель вечного двигателя
Ну это уже что-то уровня обо/сцай или даже ниже.

sageАноним Срд 23 Июл 2014 23:48:13 #309 №119118 
1406144893109.jpg

>>119105
Ясненько.

Аноним Чтв 24 Июл 2014 00:10:34 #310 №119121 

>>119118
>этот слившийся эфирный хохол
Понятненько.

Аноним Чтв 24 Июл 2014 01:24:57 #311 №119137 

>>119045
Я вижу одновременно две вспышки потому, что две молнии ударили одновременно. Одна ближе ко мне, а вторая дальше. Я вижу две вспышки, но звук по понятным причинам слышу в разное время.

Аноним Птн 25 Июл 2014 11:40:48 #312 №119242 
1406274048816.gif

Спейсач, есть реально тупой вопрос. Пикрелейтед - открытый портал с Земли на Луну (для тех, кто вдруг не играл в одноименную замечательную игрушку).
Вопрос. Можно ли так продержаться на Луне без скафандра некоторое время, обдуваясь воздухом, выносимым на тебя под давлением атмосферы всей планеты? И вообще, какую скорость способен развить такой поток воздуха при подобной теоретической "утечке"? Есть мнение, что хрен бы кто так удержался.

http://stream1.gifsoup.com/view2/2388130/portal-2-moon-shot-o.gif

Аноним Птн 25 Июл 2014 11:56:47 #313 №119247 

>>119242
> И вообще, какую скорость способен развить такой поток воздуха при подобной теоретической "утечке"?
http://ru.wikipedia.org/wiki/Закон_Бернулли

Аноним Птн 25 Июл 2014 12:17:22 #314 №119252 

>>119247
Зачем нужны коррекции? Нельзя ли сразу точно навестись на нужную траекторию?

Аноним Птн 25 Июл 2014 12:20:16 #315 №119253 

>>119252
навести можно - попасть сразу нельзя.

Аноним Птн 25 Июл 2014 13:02:02 #316 №119262 

>>119253
Почему?

Аноним Птн 25 Июл 2014 13:18:31 #317 №119263 

>>119262
ИРЛ, в смысле? Точность низкая потому что. Есть допуски на двигательную установку, систему ориентации, определение параметров орбиты. Математические модели упрощены и не учитывают всех эффектов. Присутствуют не-кеплеровские возмущения орбиты - солнечное давление, ветер, магнитное поле, остаточный драг, неравномерность гравитации, которые заранее знать сложно или невозможно (гугли что такое оскулирующая орбита, SGP, TLE и т.п.).

Все эти отклонения приходится учитывать (именно поэтому есть работа для специалистов по астродинамике и баллистике - модели крайне запутаны и сложны), но идеально это сделать все невозможно. В общем, крафты летают по точным эллипсам лишь на картинках и схемах.

А малейшее отклонение от идеального маневра сейчас даст гигантскую ошибку (тысячи километров) потом, когда ты подлетишь к другой планете.

Аноним Птн 25 Июл 2014 13:23:40 #318 №119264 

>>119263
Хотя справедливости ради - иногда и почти без коррекции попадали куда надо, правда там везения больше было (с некоторыми советскими лунными АМС, некоторыми "Венерами", и т.п.)

Аноним Птн 25 Июл 2014 18:12:19 #319 №119292 
1406297539231.jpg

Анон, не раз замечал что в высокоинтеллектуальных дискуссиях, стоит кому-либо упомянуть эфир, как у астрофизик-куна незамедлительно начинается бугурт галактического масштаба.
Чому спейсанам так неприятен эфир? Вот серьёзно анон, поясни обстоятельно за эфир.

Аноним Птн 25 Июл 2014 18:42:09 #320 №119299 

>>119292
Анон, не раз замечал, что в высокоинтеллектуальных дискуссиях, стоит кому-либо упомянуть розовых единорогов, как у биолог-куна незамедлительно начинается бугурт макроэволюционного масштаба.
Чому биологам так неприятны розовые единороги? Вот серьезно, анон, поясни за розовых единорогов.

Аноним Птн 25 Июл 2014 18:45:07 #321 №119300 
1406299507305.png

>>119299
>упомянуть розовых единорогов
Problems?

Аноним Птн 25 Июл 2014 19:20:55 #322 №119309 

>>119300
Но это не единорог. И зачем пони в спейсаче?

Аноним Птн 25 Июл 2014 19:37:25 #323 №119311 
1406302645939.png

>>119309
Ничего себе у ваз вопросики.

Аноним Птн 25 Июл 2014 22:01:56 #324 №119328 

Как на счёт космичесских кораблей/станций многогранников? В Элит были такие Взлетит?

Аноним Птн 25 Июл 2014 22:19:46 #325 №119333 

>>119328
Доставь пикчу.

> Взлетит?
Взлетит всё, что ты запульнёшь туда. Вопрос только нахуя.

Аноним Птн 25 Июл 2014 22:46:15 #326 №119334 

>>119328
Сорта дизайна

Аноним Птн 25 Июл 2014 23:26:07 #327 №119339 
1406316367641.jpg

>>119299
Спросил жи в тупых вопросах, нет, блять, надо петросянить.

Аноним Птн 25 Июл 2014 23:34:36 #328 №119341 

>>119300
> Problems?
Yes, problem.

Отсутствует наличие мультсериала со светоносным эфиром в главной роли.

Аноним Суб 26 Июл 2014 00:58:05 #329 №119357 

Посоны. Тред тупых вопросов - нате тупой вопрос.
У нас на борту был один компактный термоядерный реактор и суммарно 100 кг топлива к нему.
Сколько времени в теории (будем считать, что мы уже можем в юзабельные термоядерные реакторы) такая бандура сможет запитывать электровоз пассажирского поезда при его постоянной работе, если не брать в расчет износ / старение материалов? Плюс-минус 50 лет?

Аноним Суб 26 Июл 2014 01:08:30 #330 №119358 
1406322510890.jpg

>>119357
Астрономия, космос и паровоз на термоядерной тяге.
>>119339
Спросил жи в тупых вопросах
Извини, просто заебали толстяки, протекающие из саентача.

Кефир ВНЕЗАПНО перестал быть нужным, для объяснения явлений электромагнетизма, к тому же у него постоянно появлялись какие-то ЙОБА характеристики, и костыли, что бы оправдать его существование. И никаких пруфов, что он существует. Ни косвенных, ни прямых.

Аноним Суб 26 Июл 2014 01:26:04 #331 №119359 

>>119358
Ммм, космический электровоз?
Электровоз для движения в вакууме на колониях вне Земли?
Ладно-ладно, я обосрался

Аноним Суб 26 Июл 2014 03:09:42 #332 №119379 

>>119357
Если термоядерный реактор хоть немного похож по своим параметрам на атомный реактор - то думаю что электровоз спокойно выработает свой ресурс гораздо раньше того, как закончится топливо.
http://ru.m.wikipedia.org/wiki/%D0%90%D1%82%D0%BE%D0%BC%D0%BE%D0%B2%D0%BE%D0%B7

Аноним Суб 26 Июл 2014 06:59:58 #333 №119385 

>>119357
еравноэмцэквадрат. 9 на 10 в 18 степени джоулей. Делим на мощность электровоза примерно 10 мегаватт, получаем 28539 лет.

Аноним Суб 26 Июл 2014 07:02:13 #334 №119386 

>>119385
Только это реактор на антивеществе. У термоядерного гораздо меньше будет, дельту массы надо смотреть у топлива и получающегося выхлопа.

Аноним Суб 26 Июл 2014 08:12:23 #335 №119387 

>>119333
http://wiki.alioth.net/index.php/Icosahedron_Station_(Oolite)
http://wiki.alioth.net/index.php/Coriolis_Station_(Oolite)
http://wiki.alioth.net/index.php/Dodecahedron_Station_(Oolite)
Из элит вики.

Аноним Суб 26 Июл 2014 09:35:50 #336 №119389 

Сап, астрономы. Ничего странного не наблюдали вчера в районе 0:35 по Москве?
Наблюдал пролет группы звездообразных объектов. Внешне очень похоже на спутники, за исключением того, что это было довольно плотная группка из 12-15 штук. Летели с северо-запада на юго-восток. Сначала были довольно яркими, потом яркость начала плавно спадать, так что похоже, были подсвечены солнцем, а стало быть высота приличная, видно их должно было быть с большой площади.

Аноним Суб 26 Июл 2014 13:36:45 #337 №119409 
1406367405524.jpg

>>119389
Самолеты? Мусоросаты? (хотя мусоросаты вряд ли увидишь с земли невооруженным взглядом)

Аноним Суб 26 Июл 2014 13:52:26 #338 №119414 

Что скажете о фотонном двигателе который будет "лететь" на фотонах от термоядерной реакции ну или ИК от ядерного реактора?

Аноним Суб 26 Июл 2014 13:58:55 #339 №119415 

>>119409
Самолёты навряд ли. И визуально непохоже, и строй слишком хаотичен, кое-где очень близко друг к другу.

Аноним Суб 26 Июл 2014 17:26:15 #340 №119436 

>>119389
Могло что-то войти в атмосферу.
http://www.satview.org/lista_sat.php?cat=tle_decay
Хотя тогда ничего с похожим наклонением вроде бы не входило (хз)

Аноним Суб 26 Июл 2014 22:46:46 #341 №119493 
1406400406741.jpg

>>119389

Аноним Суб 26 Июл 2014 23:56:44 #342 №119508 

Каким был бы климат Земли без наклона планетарной оси?
Вечные льды, вплоть до средиземного моря - ведь лета то нет, короткий световой день и все такое или наоборот - растаявшие полярные шапки и субтропики в арктике - зимы ведь тоже нет, солнце равномерно греет океаны, мощные атмосферные потоки переносят тепло от вечно плавящегося экватора в полярные регионы. Как бы повлияло отсутствие наклона оси на тектоническую активность, на орбиту Луны, на стабильность климата, на наличие ледниковых периодов?

Аноним Вск 27 Июл 2014 00:31:43 #343 №119521 

Много читал про то, каких охуеннейших успехов СССР добился в исследовании Венеры, как все это вообще круто и как весь мир охуел что мы можем. Однако никак не могу понять, в чем конкретно заключалась неебическая сложность? Вся суть была в том, чтобы сконструировать зонд, способный выдержать атмосферное давление, не? На саму планету-то попасть в принципе относительно несложно ввиду наличия атмосферы, об которую можно было спокойно тормознуть. В чем же вся эпичность? Кровавое СШП вот например уже успело на орбиту каждой планеты по зонду закинуть, и на Титан сесть, а ажиотажа и близко нету. В то же время, посадкой на Венеру даже иностранцы восхищаются. В чем соль?

Аноним Вск 27 Июл 2014 00:57:23 #344 №119526 
1406408243817.jpg

>>119521
Жопой с терки съедь - поймешь, насколько тяжелая задача посадить прибор на поверхность планеты с агрессивной и плотной атмосферой, как у Венеры. Температура в 450 по Цельсию, давление, как на километровой глубине, вирга из серной кислоты. Но СССР удалось, причем аппараты даже какое-то время работали. Первое в мире изображение другой планеты. Слепящий вин, на то время.
>>119508
>Каким был бы климат Земли без наклона планетарной оси?
Немного некорректный вопрос, климат в разных частях Земли разный. Если бы наклона оси не было, то пропали бы времена года. В Москве и Питере была бы вечно осенняя погода. То днем, тепло, то заморозки ночью. 0±10 градусов, с отличием, что в Москве потеплее, а в Питере похолоднее. В районе Мурманска был бы круглогодичный дубарь, без права хотя бы на слабую на оттепель. Экватор, возможно остался бы таким, как сейчас, либо превратился бы в суровую пустыню. Тропики были бы более сухим и никаких сезонов дождей. Да вообще никаких нигде сезонов. Однако, нужно пересматривать все факторы целиком. Течения, ветра, и так далее.

Аноним Вск 27 Июл 2014 01:01:08 #345 №119527 

>>119521
Надо было ещё сесть и сколько-то проработать в этом аду. Передовая на то время аппаратура ещё. Сложная схема полета и посадки. Веги особенно сложны были: синхронное прибытие двух станций, пролет мимо кометы по дороге (!), в каждой станции по 2 зонда, сложная схема посадки.

> На саму планету-то попасть в принципе относительно несложно ввиду наличия атмосферы, об которую можно было спокойно тормознуть.
Это только в KSP так. На практике это было сильно сложней, чем мягкая посадка АМС на Луну. Дело в мелочах, как обычно.

Вообще, если перевести в современные мерки - сложность примерно как у программы исследования Марса, наверно.

> Кровавое СШП вот например уже успело на орбиту каждой планеты по зонду закинуть, и на Титан сесть, а ажиотажа и близко нету.
Есть, как же нет? На марсоходы дрочили все кому не лень, и правильно делали. Кассини-Гюйгенс это супер-охуенно, Вояджеры-Пионеры, Хаябуса от JAXA с её компьютерным сбоем и спасением, чего только не было у разных космических агентств. Сейчас вот летят Rosetta и New Horizons, вполне себе активно освещаются в медиа. Венерианская программа - тоже одна из очень успешных страниц в истории АМС. Тем более что пришлась на зарю космонавтики и пик публичного интереса, и в общем-то была на то время технически передовой.

такие вопросы надо в отдельные треды пилить

Аноним Вск 27 Июл 2014 07:21:38 #346 №119539 

>>119521
Ещё незабываем про приближенность к солнцу, излучения/вспышки, работа систем связи, наведенная радиация, все дела.

Аноним Вск 27 Июл 2014 08:27:04 #347 №119541 

Да просто, это не на марс посадить что-то, где несильно от вакуума среда для аппарата отличается. Там нужно было решить нетривиальную задачу разработать космический батискаф, чтобы ему фотоокошечки не выдавило, и чтобы он не перегрелся раньше времени потому что тепло там вообще некуда деть, можно только принять. И чтобы ни атмосферный разряд огромной мощности ни серная кислота ничего с ним не сделали.

Аноним Пнд 28 Июл 2014 03:53:16 #348 №119679 

Много где читал/слышал, что если отлететь мгновенно на много световых лет от земли и посмотреть на нее в телескоп, то можно будет посмотреть историю развития цивилизации. Вопрос в том, действительно ли это так?
Вопрос касается не истории совсем, а физики распространения света.

Дело в том что свет, отраженный от любой поверхности распространяется во все стороны, а количество этих отраженных фотонов ведь не бесконечно. Сколько фотонов полетит в мизерный участок неба, где будет располагаться когда то наш телескоп, и какого размера он должен быть чтобы хоть какие то из них поймать?

Вот представим, что через допустим пару тысяч лет наконец то смогли воплотить путешествия через червоточины. Смогут ли тогда люди разглядеть динозавров (отлететь на 100 млн световых лет и хоть что то разглядеть в супер мега телескоп), или строящиеся пирамиды (несколько тысяч световых лет), или вообще что смогут?

Если более конкретно задавать вопрос:
На расстоянии 100 млн световых лет от земли висит самый крутой телескоп, улавливающий 99% излучения, диаметром 100 метров, смотрящий на землю. Возможно ли в него разглядеть цвет глаз тираннозавра?
Если нет, то на каком расстоянии возможно?




Аноним Пнд 28 Июл 2014 04:53:46 #349 №119687 

>>119679
Удваиваю вопрос. Какое разрешение у нашей матрицы?

сейчас кто-то скажет, что под такие вопросы нужно пилить отдельные треды

Аноним Пнд 28 Июл 2014 05:33:29 #350 №119689 

>>119679
Думается, вопрос можно свести к следующему: построить 100-метровый телескоп на орбите Земли и посмотреть из него на твою мамку на Плутон или хотя бы на Марс (болты на Куриосити разглядывать). И никаких червоточин не надо.

Аноним Пнд 28 Июл 2014 16:19:03 #351 №119708 
1406549943508.jpg

КОСМОС, СУКА, КРУГЛЫЙ!

Аноним Пнд 28 Июл 2014 17:25:10 #352 №119714 

так, посоны, объясните, вселенная расширяется с постоянным ускорением или с переменным?

Аноним Пнд 28 Июл 2014 18:38:58 #353 №119717 

>>119714
Не совсем корректно воспринимать расширение пространства как кинетическое явление. Если вести речь конкретно про постоянную Хаббла, то в некоторых моделях вселенной она таки увеличивается со временем. Это можно назвать условным ускорением, но как оно изменяется пока тебе никто не ответит.

Аноним Пнд 28 Июл 2014 18:49:00 #354 №119718 

>>119714
Ускоряющимся ускорением.

Аноним Пнд 28 Июл 2014 18:56:38 #355 №119719 

>Если ускоряющееся расширение Вселенной будет продолжаться бесконечно, то в результате галактики за пределами нашего Сверхскопления галактик рано или поздно выйдут за горизонт событий и станут для нас невидимыми, поскольку их относительная скорость превысит скорость света. Это не является нарушением специальной теории относительности. На самом деле невозможно даже определить «относительную скорость» в искривлённом пространстве-времени. Относительная скорость имеет смысл и может быть определена только в плоском пространстве-времени, или на достаточно малом (стремящемся к нулю) участке искривлённого пространства-времени. Любая форма коммуникации далее пределов горизонта событий становится невозможной, и всякий контакт между объектами теряется. Земля, Солнечная система, наша Галактика, и наше Сверхскопление будут видны друг другу и в принципе достижимы путём космических полётов, в то время как вся остальная Вселенная исчезнет вдали. Со временем наше Сверхскопление придёт в состояние тепловой смерти, то есть осуществится сценарий, предполагавшийся для предыдущей, плоской модели Вселенной с преобладанием материи.
Грусть-пичаль.

Аноним Пнд 28 Июл 2014 22:59:14 #356 №119763 

>>119541
>чтобы он не перегрелся раньше времени потому что тепло там вообще некуда деть, можно только принять
А нельзя запилить технику, которая бы работала при 500С?
Это же мы, мясные мешки, такие требовательные к температурам, а чому бы не запилить корпус из вольфрамов, йоба-электронику из хитрых полувпроводников. У многих металлов же температура плавления куда выше тысячи градусов.
Почему не запилили, почему?

Аноним Пнд 28 Июл 2014 23:03:28 #357 №119764 

>>119689
У нас же атмосфера мешает.
Старый Хаббл разглядывает формы далеких галактик.
Скоро Джеймс Вебб полетит, будет планеты у соседних звезд видеть.
А если запилить обсераторию на лунной Л2 с "зеркальцем" в сто метров, то... эээ. Блин, это реально вопрос для отдельного треда.

Аноним Пнд 28 Июл 2014 23:31:06 #358 №119765 

>>119763
Потому что реальность и Лишний ствол - разные вещи.

Во-первых, среда реально ультра-агрессивная. Представь себе ванну с серной кислотой. Неслабые условия, да? А теперь представь ту же ванну, но под давлением как в кислородном баллоне для сварки (который при разрыве разносит многоэтажки), и при температуре как в муфельной печи. Все химические реакции ускоряются в дохуя раз. То что при обычных условиях никак не реагирует с серной кислотой, живет на поверхности Венеры полчаса. А йоба-химически нейтральные материалы не всегда обладают нужной прочностью и другими характеристиками. Про любые механические части вообще молчу - почти всему там приходит моментальный пиздос, а керамика какая-нибудь - представь себе фарфоровый ровер.

Веги даже парашют не использовали при финальной стадии посадки, потому что нет такого материала, который бы этот нонсенс выдержал. http://www.spek.keytown.com/rasIIIProg/5_tvorch/Space40/1998/books_98/Ladigin.htm вот хорошо пояснено за эту хуйню. Они просто падали тонули со скоростью 30км/ч, потому что атмосфера Венеры в нижних слоях находится в суперкритическом состоянии, т.е. нет четкой границы между атмосферой и океаном. А в верхних слоях постоянный ветер по 300км/ч, а в средних постоянно ебашат молнии.

Во-вторых, йоба-электроника из хитрых полупроводников работает максимум при 240 кажется C. У насы было предложение венерианского ровера, в котором работал бы холодильник Стирлинга и охлаждал бы температуро-чувствительные детали до приемлемого пиздеца (и нагревал бы окружающую среду, т.е. дополнительный челлендж). Но так всё на стадии предложений и заглохло - слишком дорого это все.

Короче, задача примерно как создание летающей подлодки - надо сочетать противоположные требования в одном аппарате. Только намного сложнее, потому что наборов условий не 2, а с десяток.

Аноним Втр 29 Июл 2014 06:31:23 #359 №119776 

>>119765
Похоже, это был вопрос для отдельного треда.

Аноним Втр 29 Июл 2014 11:47:22 #360 №119791 

Дико проиграл с манямирка. А если у нас спросить мимокрокодилов, ответы такие же печальные будут?

Аноним Втр 29 Июл 2014 12:06:08 #361 №119796 

>>119791
Да.

Аноним Втр 29 Июл 2014 13:23:20 #362 №119802 

>>119791
Очень простые вопросы. А вот если хоть немного усложнить, например, предложить перечислить планеты по порядку от Солнца и далее, то 98% не смогут ответить, я проверял на знакомых. Такая вот хрень бля.

Аноним Втр 29 Июл 2014 14:03:41 #363 №119804 

>>119802
Одни кретины кругом же. Не то что планеты перечислить - столицу Новой Зеландии назвать не могут, ни начертить куб в перспективе, да даже назвать любую породу овец, не могут описать диод, не знают нот, не разделят многочлен на двучлен, не вспомнят ни одного известного шахматиста, да что там, в сами шахматы играть не умеют, сколько у человека костей, где живут белые медведи, ни одного стихотворения Блока наизусть не помнят, даже названия, убунту не поставят, основные правила употребления артиклей в английском языке не помнят, Мадоки не видели, высочайшего водопада не назовут, не знают первооткрывателя Антарктиды, императора, основавшего Петербург не знают, ничего не знают, не помнят, не видели. Идиоты. Биотопливо.

Аноним Втр 29 Июл 2014 14:09:30 #364 №119805 

>>119791
>все ответили US
Спасибо, снабдил меня бугуртом на день.

Аноним Втр 29 Июл 2014 14:24:19 #365 №119807 

>>119791
Естественно, как и везде

Аноним Втр 29 Июл 2014 14:39:13 #366 №119809 

>>119804
>Одни кретины кругом же. Не знают, ничего не знают, не помнят, не видели. Идиоты. Биотопливо.

Абсолютно согласен. Такой биомусор не нужен в современном мире. Специализация - удел насекомых. А настоящего человека, прежде всего, отличает широкий кругозор.

Аноним Втр 29 Июл 2014 16:27:38 #367 №119833 
1406636858530.png

почему не принята сферическая модель вселенной? почему она вообще практически не пользуется вниманием? всякие говнотрубы, кольца и прочее дерьмо. просто для меня всегда было слишком очевидно, что она расширяется во всех направлениях, а все космические объекты находятся на поверхности сферы, из-за чего собсна они удаляются друг от друга, из-за чего нельзя достигнуть конца вселенной, и прочая логически всплывающая хуйня. яннп

Аноним Втр 29 Июл 2014 16:35:51 #368 №119836 

>>119833
Это инфографика же, ещё и хуёвая какая-то, так и не вкурил, как её понимать.

Аноним Втр 29 Июл 2014 16:58:15 #369 №119840 

>>119765
Спасибо, хорошее пояснение.
мимо

Аноним Втр 29 Июл 2014 17:03:05 #370 №119841 

>>119804
Схоронил. Вассерман залогинитесь

Аноним Втр 29 Июл 2014 17:07:22 #371 №119843 

>>119791
Поэтому ваш космос не нужен обывателю.

Аноним Втр 29 Июл 2014 17:13:40 #372 №119845 

>>119805
Поцреот забургутил. Что за двойные стандарты? Рашкованин может и мурриканец тоже ответит кто высадился на Луну и в каком году, сколько раз всего? А ведь есть люди, которые верят, что это мистификация, вот где баттхерт.

Аноним Втр 29 Июл 2014 17:25:46 #373 №119848 
1406640346766.jpg

Анон, а что будет, если одна нейтронная звезда упадет на другую?

Аноним Втр 29 Июл 2014 17:33:43 #374 №119850 

>>119848
Получится чёрная дыра твоей мамки

Аноним Втр 29 Июл 2014 17:38:37 #375 №119851 

>>119848
>>119848
Слияние нейтронных звезд. С образованием черной дыры..
Алсо, где-то читал статейку, в которой приводились доводы в пользу того, что тяжелые элементы, например золото и выше не могут образовываться при взрыве сверхновых в таком количестве, а образуются такие элементы именно при столкновении двух нейтронных звезд.
https://www.sciencenews.org/node/16219

Аноним Втр 29 Июл 2014 21:53:00 #376 №119912 

>>119851
О, круто! Но, все таки банально...
Это как-будто ответ на все вопросы.

Аноним Втр 29 Июл 2014 22:47:25 #377 №119951 

>>119912
>Но, все таки банально
Ну там много интересного происходит, например гамма-всплеск чудовищной силы.
А таком танце двух близких НЗ гравитационные волны взъебывают пространство с такой силой, что эти волны, попытаются задетектить с помощью проекта LISA.

Аноним Срд 30 Июл 2014 04:28:46 #378 №120039 

а существуют ли солнечные системы но не со звездой по середине а с черной дырой? какие профиты если такие есть?

Аноним Срд 30 Июл 2014 14:15:48 #379 №120100 

>>120039
Планеты вряд ли cмогут пережить взрыв сверхновой, так что планеты, обращающиеся вокруг ЧД, скорее всего захвачены из вне, а не являются остатками своей некогда существующей планетарной системы.
Профитов в этом случае никаких, ибо видеорелейтед.

Аноним Срд 30 Июл 2014 14:53:40 #380 №120102 

>>120100
извне

very slow fix

Аноним Чтв 31 Июл 2014 23:10:19 #381 №120335 

Могут ли быть в облаке Оорта крупные объекты с массой сравнимой с массой Земли?

Аноним Чтв 31 Июл 2014 23:22:31 #382 №120351 

>>120335
> сравнимой с массой Земли
По современным представлениям считается, что крупные каменистые планеты не могут образовываться сильно дальше снеговой линии воды (для солнечной системы это 5 ае). А намеков на существование газовых гигантов там пока что не зарегистрировано, за исключением недавней работы, которую линканули в соседнем треде.

Аноним Птн 01 Авг 2014 13:45:48 #383 №120412 
1406886348343.jpg

Анон, а сколько времени понадобится, что бы заморозить Землю? На сколько потребуется убрать Солнце, что бы гарантировано выпилить все живое?

Аноним Птн 01 Авг 2014 14:04:58 #384 №120417 

>>120412
На дне останется геотермальная жизнь.

Аноним Птн 01 Авг 2014 14:08:38 #385 №120418 

>>120417
Ну а человеки как быстро отъедут?

Аноним Птн 01 Авг 2014 17:21:56 #386 №120443 

>>120418
В Зеон? В течении года.
>>120417
Два ведра воздуха этому адеквату.

Аноним Птн 01 Авг 2014 17:45:38 #387 №120446 
1406900738287.jpg

>>116435
Пространство нельзя сжать! Что вы несете, еретики? Пространства как такового не существует - это просто абстракция, которую придумали люди для того, чтобы измерять положение одних объектов относительно других. МИНЯ АЖ ТРИСЕТ, НЕУЖЕЛИ ФИЗИКИ ТОКИЕ ТУПИЕЕЕЕ!!!!11йк231е532

Аноним Птн 01 Авг 2014 17:58:51 #388 №120447 

>>120446
Как там в 19 веке?

Аноним Птн 01 Авг 2014 18:09:39 #389 №120448 

>>120418
Какая степень отъезжания нужна? Единицы в уникальных условиях, вероятно, будут жить и функционировать до конца жизни. Сомневаюсь, что самоподдерживающаяся система где-то будет создана, так что человечество с годами вымрет. Куда тяжелее избавиться от жизни как таковой. Что-то будет столетиями существовать на тающих ресурсах и внутри остывающих структур, что-то и вовсе будет жить полноценной микробной жизнью - какая-нибудь сероводородная хуерга в подводных вулканах и прочем. А так, наверное, даже сутки без солнца приведут к тяжелой экологической катастрофе и кризису, но без особых жертв. За неделю, вероятно, вымерзнет 99% людей, но всё равно будет кому восстановиться в уровне жизни лет через 30. Но мне лень проводить анализ, так что останусь кухонным аналитиком.

Аноним Птн 01 Авг 2014 21:26:02 #390 №120466 

>>120412
http://what-if.xkcd.com/49/

Аноним Птн 01 Авг 2014 21:37:09 #391 №120468 

>>120446
Да они тупые, им господин запретил говорить о эфире под страхом анафемы и теперь они вынуждены сжимать не эфир, а пространство.

Аноним Птн 01 Авг 2014 22:33:55 #392 №120473 

>>120446
Почитай Снегова лалка

Аноним Вск 03 Авг 2014 16:23:57 #393 №120733 
1407068637577.jpg

Как минимизируют потери воздуха в EVA? Судя по всему, целые отсеки или даже весь интерьер лендера теряют несколько килограмм воздуха всякий раз, когда герметичность нарушается. Его бережно выкачивают или же просто восполняют из запасов, обновляемых с земли?

Аноним Вск 03 Авг 2014 17:47:52 #394 №120741 

>>120733
Его выкачивают, у лендеров запланированное количество выходов, основные траты не на заполнение обьема, а на восстановление кислорода, вообще кислородная проблема вторичная в дальних экспедициях по сравнению с проблемой жратвы, замкнутый цикл восстановления кислорода из углекислого в течении суток умещается в наспинном рюкзаке вместе с климат контролем и связью, почти замкнутый цикл воды на мире/МКС уже некислая установка с внешней минерализацией, замкнутая биосфера с едой на группу космонавтов в марсианском полёте займёт неебическое пространство, даже при минималистичном исполнении на водрослях/бактериях, и затраты на шлюз, даже при отброшенной возможности выделять кислород из атмосферы марса(там мало его, но он есть) будут ничтожны по массе.

Аноним Вск 03 Авг 2014 17:58:52 #395 №120743 

Вот я смотрю бибисишную документалку про полеты по всей солнечной системе. Они включили магнитное поле, и у них началось северное сияние. Я так понимаю, что эта документалка может быть менее твердой, чем Армагеддон, но всё же, что там может сиять такое? Вакуум же.

>>120741
Спасибо.

Аноним Вск 03 Авг 2014 18:28:27 #396 №120747 
1407076107901.jpg

>>120743
Ионизированные частицы чего-либо, например. Взяться им там неоткуда, ионизироваться нечем. В данном случае это корабль В ПРЕДСТАВЛЕНИИ ХУДОЖНИКА, не обращай внимания на ляпы.

Аноним Вск 03 Авг 2014 18:39:16 #397 №120749 
1407076756568.jpg

>>120747
У BBC-то консультанты стопудово были нормальные. Это просто, похоже, ни что иное, как YOBA в первоначальном смысле этой аббревиатуры. Твой пик из этого же фильма, видимо, постер к нему.

А вообще, весь фильм сделан по одному лекалу: начало, внезапное страшное обстоятельство, превозмогание, развязка, в ЦУПе хлопают и свистят. Повторить около 10 раз за 80 минут.

Аноним Вск 03 Авг 2014 18:58:29 #398 №120753 
1407077909106.jpg

Чем и для чего размечен на секторы с текстом купол? Как будто маркером.

Аноним Вск 03 Авг 2014 20:58:10 #399 №120777 
1407085090263.jpg

>>120753
Это где ты пикчу эту взял?
Настоящий модуль Купол выглядит вот так. Он обёрнут защитным чехлом.

Аноним Вск 03 Авг 2014 20:59:48 #400 №120778 
1407085188453.jpg

>>120777
Как видишь никаких марок нет. Возможно какая-то разметка и была нанесена на заводе, чтобы ориентироваться сборщикам, но на МКС она скрыта под оболочкой.

Аноним Вск 03 Авг 2014 22:46:57 #401 №120784 

>>120777
Из документалки, это фейкокупол. Просто стало интересно, нахуя они это сделали, не кроется ли в этих надписях какой-то фундаментальный секрет космонавтики.

Аноним Пнд 04 Авг 2014 12:02:10 #402 №120843 
1407139330329.jpg

В космосе межзвездном пространстве темно? Ну как в темной комнате - вытянутую руку не видно?

Аноним Пнд 04 Авг 2014 14:20:45 #403 №120851 

>>120843
Смотря где. Если звезда светит прямой наводкой (т.е. между ней и тобой ничего нет) - то можно и глаза пожечь, и кожу на роже (от чего у космонавтов есть светофильтр на шлеме, хотя долго смотреть на звезду все равно не советуют). Если же ты над темной стороной планеты (или просто тебя загораживает, скажем, солнечная панель) - ничего без фонаря ты не увидишь.

Аноним Пнд 04 Авг 2014 19:26:10 #404 №120883 

>>120843
> межзвездном пространстве
В межзвездном или просто в тени - темно. Руки видно. Как в деревне в звездную безлунную ночь, чуть ярче.

Аноним Втр 05 Авг 2014 01:21:51 #405 №120917 

Выручай космочан! Читаю Азимова "Немезида" и так и не могу врубиться, что такое есть ПАРАЛЛАКС. Объясните кто-нить доходчиво. с меня ничего статью вики не понял ибо туп

Аноним Втр 05 Авг 2014 01:29:16 #406 №120918 
1407187756010.gif

>>120917
> что такое есть ПАРАЛЛАКС

Аноним Втр 05 Авг 2014 08:17:49 #407 №120952 

>>115935
Если на орбите диск 100м вертится с околосветовой скоростью (допустим он бесконечно крепкий), то время на краю диска - замедляется наиболее. Замедленное время - в чем локализовано? В материале? Гравитации диска? Где его граница?

Аноним Втр 05 Авг 2014 09:15:32 #408 №120954 

>>120917
Это слово слишком умное для обозначения тупой хуеты.
Если ты будешь смотреть на палец на вытянутой ручечке поочередно одним и другим глазиком но не шоколадным, палец будет оказываться на фоне разных удаленных предметов. Расстояние между глазиками - это база.
Параллактическое смещение - тоже самое, только масштаб другой. Острономы поочередно смотрят на близкую звездочку то с одной стороны орбиты, то с другой. Зимой и летом, например.
От этого им кажется, что звездочка в течении года перемещается туда-сюда относительно других, более лучших далеких звезд.
Таким нехитрым способом они и смекают, какие звездочки поближе, а какие - подальше.

Аноним Втр 05 Авг 2014 09:31:44 #409 №120955 

>>120952
> допустим
Нет, не допустим.

Аноним Втр 05 Авг 2014 09:59:24 #410 №120958 

>>120955
Да ладно, точки поверхности быстрых НЗ крутятся вполне себе релятивистски, у миллисекундных пульсаров - до четверти световой.

Аноним Втр 05 Авг 2014 11:02:41 #411 №120969 

Существуют ли планеты далеко за пределами галактик(в войдах например)?

Аноним Втр 05 Авг 2014 11:03:47 #412 №120970 
1407222227610.jpg

>>120955
Хорошо, пусть он крутится с вполне себе респектабельной скоростью, неспособной разорвать его на части. Тем не менее, его "личное" время плавно замедляется от центра к краю. Мы висим неподвижно возле края диска. Как близко мы поднесем, или погрузим в материю диска тонкий молекулярный щуп что-бы личное время молекул на его конце - то же замедлилось?

Аноним Втр 05 Авг 2014 16:32:59 #413 №120994 
1407241979350.jpg

>>120970
>>120970
Благодарю, затупил в случае с диском. В звезде гравитация держит вещество. Время распространяется в гравитационном поле и не имеет границы влияния. Доказательство - эквивалентность релятивистских эффектов скорости и гравитации. Сам спросил и сам ответил.

Борды по теме:
>>118639
>>104172
>>116391

Аноним Втр 05 Авг 2014 17:23:43 #414 №120998 

>>120412
http://www.youtube.com/watch?v=7DAjOW7u5-Y

Аноним Втр 05 Авг 2014 20:36:47 #415 №121025 

>>120969
Если столкнулись голлактеки, то запросто звёздные системки могло вышибить. Но системки надо бы ещё от собсно звёзд отлучить. Стало быть, во время свистопляски столкновения голлактек, звёздные системы сталкивались\проходили достаточно близко, чтоб нарушить орбиты плонет и они улетели нахуй в ВОЙД БЛЯДЬ ЧОРНЫЙ ХОЛОДНЫЙ МИЛЛИАРД СВЕТОВЫХ ЛЕТ ПУСТОТЫ НИХУЯ НИКАКОГО ТУШНЯКА НЕ НАПАСЁШЬСЯ ПРОЛЕТЕТЬ ТАКОЙ ДА И ХУЙ ПРОЛЕТИШЬ БАБАЙКА ИЗ ТЁМНОЙ ЭНЕРГИИ ТЕБЕ ПЛОНЕТЕ ЖОПУ ОТКУСИТ ПИЗДА ВСЕ ЗАМЁРЗЛИ БЕЗ ОТОПЛЕНИЯ ТОЛЬКО ТРУПЫ ЗАМЁЗРШИЕ В ПУСТОЕ НЕБО ПЯЛЯТСЯ ГЛАЗНИЦАМИ ПУСТЫМИ "ЧОМУ Я ХОХОЛ"

Аноним Втр 05 Авг 2014 23:36:20 #416 №121051 
1407267380944.jpg

Шарился по википедии и наткнулся на пикрелейтедю
https://ru.wikipedia.org/wiki/%D0%9B%D1%83%D0%BD%D0%B0-12
Как оно вообще на Луну-то упало? Почему с орбиты сошло?

Аноним Втр 05 Авг 2014 23:44:57 #417 №121054 

>>121051
Неравномерность гравитации. Изначально перицентр был около 100км, потом стал резко падать, и его пришлось поднимать. И все равно ебнулось.

Аноним Втр 05 Авг 2014 23:58:10 #418 №121058 

>>121054
Странно как. У Луны форма не идеальная что ли? С чего вдруг неравномерность?

Аноним Срд 06 Авг 2014 00:18:17 #419 №121060 

>>121058
Распределение масс внутри Луны неодинаковое на видимой и обратной стороне. Неравномерность ее гравитации в несколько раз больше земной из-за этой хуйни, плюс локальные концентрации массы на поверхности, поэтому неопределенность орбиты довольно велика. Проход LADEE в 300м от поверхности на последнем витке это вообще чудеса эквилибристики.

Аноним Срд 06 Авг 2014 00:23:22 #420 №121063 
1407270202306.jpg

>>121058
>У Луны форма не идеальная что ли. С чего вдруг неравномерность??

Да у тебя же МАСКОН

Аноним Срд 06 Авг 2014 00:41:33 #421 №121066 

>>121063
>>121060
Нихуево так.

Аноним Чтв 07 Авг 2014 11:58:42 #422 №121307 

Поток электронов может создать тягу? Если да, то можно ли запилить чисто электрический двигатель(эдакие "молнии" которые разгоняют корабль)?

Аноним Чтв 07 Авг 2014 14:51:41 #423 №121322 

>>121307
Импульс-то электрону придать можно, проблема в том, чтобы заставить его двигаться куда-либо кроме как между двумя электродами. (и в эффективности получившейся ебалы)

Аноним Чтв 07 Авг 2014 15:06:33 #424 №121326 

Космачи! Какие есть способы создания искуственной гравитации на кораблях? Из вики прочитал:
Роторный - значит гравитация будет только в на внутренней поверхности тора? (как в фильме одиссея)
Линейный - то есть выходит что если я набираю скорость меня как в машине/самолете к креслу тянет? Но когда я достиг нужного ускорения - все типа невесомость?
Масса - фактически если я создам термояд на корабле, то у меня будет сфера с гравитацией? А насколько надо много массы и соответсвенно энергии впихнуть туда чтобы было 1 земная?
И собственно антигравитаторы - я так понимаю чистая магия?
больше я так понимаю нету?

Аноним Чтв 07 Авг 2014 15:11:30 #425 №121327 

>>121326
>Какие есть способы создания искуственной гравитации на кораблях?
Только один, при помощи инерции. Больше никаких.

> Но когда я достиг нужного ускорения - все типа невесомость?
Можно перевернуться и тормозить в противоположном направлении, например. И да, ты наверно имел в виду "нужной скорости".

> И собственно антигравитаторы - я так понимаю чистая магия?
По современным представлениям - да.

Аноним Чтв 07 Авг 2014 15:17:08 #426 №121329 

>>121326
>А насколько надо много массы и соответсвенно энергии впихнуть туда чтобы было 1 земная?
6*1024кг (6 триллионов миллиардов тонн)

Аноним Чтв 07 Авг 2014 15:37:00 #427 №121333 
1407411420545.jpg

>>121329
Почему такая разница?
Я так понимаю, чтобы у меня был термояд дающий гравитацию и небыло ебанной черной дыры на борту, мне нужно чтобы была масса (которая я так понимаю лучше брать из самых тяжелых элементов?) меньше критической для образования ЧД? Или тут важен радиус? Его можно как то регулировать?

Аноним Чтв 07 Авг 2014 16:01:24 #428 №121335 

>>121329
вообще мне нужно понять какой размер будет у ТЯ энергоустановки на космическом корабле, чтобы понять размер этого корабля. То есть насколько допустимо сжимать эти 6 триллионов миллиардов тонн в определенный радиус?

Аноним Чтв 07 Авг 2014 16:29:21 #429 №121337 

>>121335
>То есть насколько допустимо сжимать эти 6 триллионов миллиардов тонн в определенный радиус?
Если ты имеешь ввиду сжимать до не превращения в ЧД, то
>Гравитационный радиус обычных астрофизических объектов ничтожно мал по сравнению с их действительным размером: так, для Земли r_g = 0,884 см, для Солнца r_g = 2,95 км
С другой стороны масса Земли, сжатая хотя бы в 100-метровый шар уже на своей поверхности будет иметь гравитацию, не совместимую с жизнью. Не говоря уже о других параметрах.

Аноним Чтв 07 Авг 2014 16:33:42 #430 №121338 

>>121335 Размер бейсбольного мячика для Земли насколько я помню. Но это чистая фантастика потому что добиться такой плотности анрил. Проще натаскать столько же обеднённого урана

Аноним Чтв 07 Авг 2014 16:34:54 #431 №121339 

>>121337
>С другой стороны масса Земли, сжатая хотя бы в 100-метровый шар уже на своей поверхности будет иметь гравитацию, не совместимую с жизнью. Не говоря уже о других параметрах.
Можно банально "отдалится" от этого шара. Что то вроде сферы Дайсона вокруг него. Жить на внешней поверхности

Аноним Чтв 07 Авг 2014 17:05:45 #432 №121344 

>>121339
Но нахуя , ведь можно просто напихать меньше массы. Допустим мы имеем скопление массы в определенном компактном объеме. Чтобы в 100 метрах от центра объекта гравитация была 1g+/- потребуется масса 1,469*10^15 кг, если не наебался с расчетами.

Аноним Чтв 07 Авг 2014 17:07:47 #433 №121345 
1407416867861.jpg

>>121339
Вах!А где я буду Морию делать тогда, шайтан?

Аноним Чтв 07 Авг 2014 20:39:10 #434 №121380 
1407429550114.png

>>121333
> Почему такая разница?
Чтобы получить земную гравитацию, тебе надо нагнать массы столько же, сколько есть у Земли. Все просто. Причем тут масса Солнца?

> термояд дающий гравитацию
Волны, перекатывающиеся стремительным домкратом.
Мечущиеся стрелки осциллографа.
Сверла, сверлящие отверстия в макаронах.
Установка по сжижению вакуума.
...
наркоман штоле, сука?

Аноним Чтв 07 Авг 2014 20:41:18 #435 №121381 

>>121326
> если я создам термояд на корабле, то у меня будет сфера с гравитацией?
Если Чингис-хан сожрет полтора ведра говна, полетит ли крокодил в Германию?

Аноним Чтв 07 Авг 2014 21:21:34 #436 №121391 
1407432094564.png

>>121326
Тащемта можно конбинировать
Взять луну , сжать ее до размера астероида например и разгонять с ускорением 8,19 м/с2 Тогда вперде буде 9,81 а на заде 6,57 м/с2 ИЧСХ заднее ускорение будет направлено вперед по ходу, евпочя

Ну и совсем хорошо, ежели эту луну размером с астероид раскатать в бублик, то есть тор. Тор то есть бублик хорошенько раскрутить, только чтобы не распидорасило и тоже отправлять в полет. Другое дело, что корабль станет похож на печальную рыбу-солнце, которая может плыть только в одном направлении, ибо гироскопический эффект поворачивать не даст.
бледь гравиполе вращающегося самогравитирующего тора это же чистый омск

тысячи людей и инопланетян на двух с половиной миллионах тонн вращающегося металла среди бездны космоса. Порой здесь небезопасно, но это наша последняя надежда на прочный мир в голлактике

Аноним Чтв 07 Авг 2014 21:56:10 #437 №121408 

Какого диаметра будет шарик из метал. водорода с гравитацией = лунной? Можна ли будет безопасно ходить по его поверхности?

Аноним Чтв 07 Авг 2014 22:55:53 #438 №121426 

>>121408
>Какого диаметра будет шарик из метал. водорода с гравитацией = лунной?

Плотность твердого водорода в 40 с хуем раз меньше плотности Луны. Вангую, что шарик будет довольно большим по диаметру.

>Можна ли будет безопасно ходить по его поверхности?

Металлический водород вообще нихуя не безопасная хуита. При температуре около 7-14 Kельвинов, водород нужно сжать под давлением в 300 ГПа, до металлических свойств. Если водород будет более комфортной температуры, скажем в 230 Kельвинов, то что бы сжать его до металлических свойств потребуется 450 ГПа.
Я хуй знает, как ты будешь ходить под давлением, больше атмосферного в 4.5 миллиона раз.

Аноним Чтв 07 Авг 2014 23:32:29 #439 №121429 
1407439949074.jpg

Ребята , скиньте пожалуйста запись переговоров погибшей ,русской космонавтки. Еще вроде итальяшки записали

Аноним Чтв 07 Авг 2014 23:32:35 #440 №121430 

Очень важный вопрос! Короче у меня спор с одним хуем. Он загадал загадку: что на луне (похуй где, главное чтоб без атмосферы) быстрее упадет с одной высоты, слон или перо. Он пиздит что они вместе упадут, одинаково. Я говорю что слон.
Кто прав и почем?

Аноним Чтв 07 Авг 2014 23:34:38 #441 №121431 

>>121429
Чиво блять?! Впервые слышу про это, доставь стори!

sageАноним Чтв 07 Авг 2014 23:35:19 #442 №121432 

>>121429
Записали они какую-то шлюху, не имеющую к космосу отношение.

sageАноним Чтв 07 Авг 2014 23:37:17 #443 №121433 

>>121431
Это фейк древний, якобы какие-то омичи смогли перехватить запись с ЦУПом, что вообще охуеть, тем более что там идет шифрование. Да и пусков никаких не было, не говоря уже о мифических погибших космонавтках. Типичная утка для новостей, короче.

Аноним Чтв 07 Авг 2014 23:39:21 #444 №121434 

>>121430
Ускорение свободного падения какое на Луне? А теперь объясни, схуяли для разных объектов оно внезапно разным должно оказаться? Это ещё в школе на уроках физики рассказывают же, блеать.

Аноним Чтв 07 Авг 2014 23:41:49 #445 №121435 

>>121432
>>121433
Ну доставьте же вы Христа ради.
Ну или подобные , не фэйковые.

Аноним Чтв 07 Авг 2014 23:42:35 #446 №121436 

>>121434
Так я нихуя не понял, они вместе упадут, или что-то раньше?

Аноним Чтв 07 Авг 2014 23:58:06 #447 №121443 

>>121436
Я же пишу - ускорение одинаковое! Как же ещё они могут упасть, если не одновременно.

Аноним Птн 08 Авг 2014 00:02:08 #448 №121444 

>>121443
Алсо для наглядности: если на Земле бросить перо в вакуумной камере, оно плюхнется на пол так же, как какой-нибудь кирпич.

Аноним Птн 08 Авг 2014 00:08:29 #449 №121446 

>>121436
Ну вообще в случае идеально чистого физического эксперимента слон таки упадет раньше. Масса слона очевидно больше, поэтому по закону всемирного тяготения сила гравитационного взаимодействия между телами системы слон-Луна будет выше, нежели между телами системы перо-Луна. Однако текущей чувствительности приборов не хватит, чтобы уловить эту разницу, поэтому тот хуй либо берет тебя на понт, либо тупой уебан.

Аноним Птн 08 Авг 2014 00:13:28 #450 №121449 

>>121446
Кто то из астронавтов проводил такой опыт на Луне - он одновременно бросал молоток и бумажку и они падали с равной скоростью

Аноним Птн 08 Авг 2014 00:14:45 #451 №121451 

>>121446
>>121443
Спасибо этим боярам

Аноним Птн 08 Авг 2014 00:22:32 #452 №121454 

>>121449
Ну так скомкай бумажку, так они и на Земле одновременно упадут. Разговор не за парусность тела, а за закон всемирного тяготения. Массивный объект падает быстрее, хоть и настолько незначительно, что современные приборы этого не засекут.

Аноним Птн 08 Авг 2014 00:51:10 #453 №121456 

>>121449

Аноним Птн 08 Авг 2014 00:55:40 #454 №121457 

>>121391
У меня от тебя BABYLON 5

Аноним Птн 08 Авг 2014 09:03:25 #455 №121491 

>>121456
Надо же, а я вчера так и не нашел его.

Аноним Птн 08 Авг 2014 10:04:38 #456 №121492 

>>121454
современные знавчит не засекут, а ты значит засекаешь? из будущего чтоли дохуя?

Аноним Птн 08 Авг 2014 11:57:50 #457 №121497 

>>121426 Окей, а если шарик из железа/обеднённого урана?

Аноним Птн 08 Авг 2014 12:30:06 #458 №121498 

test

Аноним Птн 08 Авг 2014 12:38:42 #459 №121499 

>>121446
>масса слона больше
Единственная разница в скоростях падения слона и пера возникнет лишь за счет того что планета подастся в сторону падающего на нее предмета с разным ускорением. Да и то, орбиту планеты это не изменит ни на сколько, потому что слон и планета являются частью одной системы - то есть она в момент поднимания слона в воздух немного отодвинется от центра масс системы планета-слон а потом во время его падения приблизится обратно.

Аноним Птн 08 Авг 2014 16:15:35 #460 №121517 

>>121492
Я опираюсь на теорию. Ну и вообще любой разговор о падающих слонах на Луне - теория.

Аноним Птн 08 Авг 2014 17:24:53 #461 №121522 

>>121497
Может тогда сразу из осмия?
Тогда шар равной массы буде в 1.88 раза меньше по радиусу.
Правда в этом случае гравитанция ускорение свободного, если чо на его поверхности будет больше лунной из-за меньшего расстояния между телами.
Так что шар нужно делать еще меньше. Насколько меньше - чота я туплю, не могу придумать как пащитать на пальцах. похоже там дифуры

Аноним Птн 08 Авг 2014 17:29:57 #462 №121523 

>>121522
А да, шары равной массы из железа и обогащенного урана меньше луняшки по диаметру в 1.32 и 1.78 раза соответственно.

Аноним Птн 08 Авг 2014 21:39:10 #463 №121570 

Есть значит комета чурова-герасемени. Достаточно ли там гравитации что бы космонафт не улетел в открытый космос от легкого толчка?

Аноним Птн 08 Авг 2014 21:59:25 #464 №121574 

>>121570
Отвечает филиал /b/ Европейского Космического Агентства.
http://blogs.esa.int/rosetta/2014/08/04/could-you-jump-onto-a-comet/

Там вопрос был про другое немного, но осилив статью, можно сказать с уверенностью, что от легкого толчка не улетит

Аноним Птн 08 Авг 2014 22:00:55 #465 №121575 

>>121570
Если ты про комету Чурюмого геросемена, то недостаточно.

вторая комическая порядка полметра в секунду

Аноним Пнд 11 Авг 2014 00:10:51 #466 №121877 

Тред не читал.
Почему на Луну больше не запускают людей? Планируются ли в ближайшем будущем отправки людей туда?
И еще.
От нечего делать вчера посмотрел РЕН-ТВ и там говорили про какую-то планету, на которой вероятно есть жизнь и сказали, что узнать это удастся уже в 2014 году. Можно подробнее об этой планете и проекте, если кто знает?

Аноним Пнд 11 Авг 2014 01:11:03 #467 №121895 

>>121877
>Почему на Луну больше не запускают людей?
Дорого. Нехуй было там делать.
>Планируются ли в ближайшем будущем отправки людей туда?
Рогозин сим бредит, чем закончится пока не ясно. Может китайцы еще подтянуться.
>посмотрел РЕН-ТВ
>там говорили
>Можно подробнее
Вот у них и узнай. Тут никто про бредни РенТВ из грибных отваров не в курсе.
Есть жизнь на Марсе - нету жизни на Марсе. Это науке неизвестно. Наука пока не в курсе.

Аноним Пнд 11 Авг 2014 05:31:35 #468 №121945 

>>121877
> Можно подробнее об этой планете
https://ru.wikipedia.org/wiki/Земля

Аноним Пнд 11 Авг 2014 14:40:22 #469 №121986 

http://news.discovery.com/space/powering-a-starship-with-a-black-hole-engine-140114.htm взлетит? Насколько это опасно?

Аноним Пнд 11 Авг 2014 23:08:06 #470 №122047 

Анон, поясни за нейтронные звёзды. Дело в том, что нейтронная звезда вырожденная? Да?

Аноним Пнд 11 Авг 2014 23:17:34 #471 №122049 

>>121986
Хуйня какая-то омская.

>>122047
Какое дело?

Аноним Пнд 11 Авг 2014 23:26:04 #472 №122051 

>>122047
>Дело в том, что нейтронная звезда вырожденная? Да?
Дело в том, что НЗ состоит из вырожденного вещества.
Это такая хуйня, когда из-за гравитации частицы так близко придавлены друг к другу, что электроны вдавливаются в протоны, от чего те становятся нейтронами. Все вещество является плотным комком из нейтронов, за исключением тонкого верхнего слоя, состоящего из тяжелых атомных ядер и электронов.

Аноним Втр 12 Авг 2014 22:54:02 #473 №122139 

Что то умные все стали(

Аноним Втр 12 Авг 2014 23:37:21 #474 №122141 

>>122139
Скучаешь по переводчику штоле?
Лучше бы спросил какую-нибудь хуйню

Аноним Срд 13 Авг 2014 01:04:45 #475 №122143 

>>122141
Расскажи про переводчика.

Аноним Срд 13 Авг 2014 03:12:56 #476 №122148 

>>122143
Не положено.
Сьеби нахуй, в обо/сцай.

Аноним Срд 13 Авг 2014 16:14:00 #477 №122183 

>>122141
Нет, просто это мой любимый тред.

Аноним Чтв 14 Авг 2014 01:45:27 #478 №122235 
1407966327165.jpg

Анон, Hubble может разрешить отпечаток Нейла Армстронга?

Аноним Чтв 14 Авг 2014 07:32:05 #479 №122244 

>>122235
Нет конечно.
Даже с орбиты луняшки проблематично разглядеть.

Аноним Чтв 14 Авг 2014 09:03:26 #480 №122245 

>>122235
Кроме мыла он ничего не увидит. Хаббл не умеет следить за быстрыми объектами технически.

Аноним Чтв 14 Авг 2014 11:53:55 #481 №122252 
1408002835351.jpg

Где здесь место Аллаху?

Аноним Чтв 14 Авг 2014 12:18:26 #482 №122254 

>>122252
Аллах везде.

Аноним Чтв 14 Авг 2014 12:30:22 #483 №122257 

>>122254
Т.е. я - тоже аллах?

Аноним Чтв 14 Авг 2014 12:36:08 #484 №122259 

Какова вероятность того что камень в моём дворе - Больцманский мозг?

Аноним Чтв 14 Авг 2014 13:05:48 #485 №122261 

>>122257
Ты раб божий, но в тебе есть частичка Аллаха.

Аноним Чтв 14 Авг 2014 13:38:03 #486 №122266 

>>122259
https://ru.wikipedia.org/wiki/Больцмановский_мозг

Аноним Чтв 14 Авг 2014 16:56:14 #487 №122285 
1408020974289.jpg

>>117266
А я б не прочь глянуть такое. Есть чо?

Аноним Птн 15 Авг 2014 01:39:09 #488 №122345 

Если черная дыра захавает планету, то её масса увеличится на массу нерадивой космической путницы? Или же Хуй Его Знает?

Аноним Птн 15 Авг 2014 02:24:51 #489 №122346 

>>122345
>Если черная дыра захавает планету, то её масса увеличится на массу нерадивой космической путницы?
Да

Аноним Птн 15 Авг 2014 04:26:51 #490 №122347 

>>122346
нет.

ЧД не "хавает" планеты. Она их разрывает своим грав полем до элементарных частиц, которые уже не поддаются влиянию гравитации, и разлетаются по вселенной. ЧД это просто шредер вселенной, уничтожитель лишнего говна. ЧД не накапливает материю, она ее уничтожает, делит на ноль.

Аноним Птн 15 Авг 2014 04:35:51 #491 №122348 
1408062951222.jpg

>>115935
Аноны, как считаете, есть ли живые организмы у планет звезды Глизе 581 ?

Аноним Птн 15 Авг 2014 04:55:57 #492 №122350 

>>122348
бактерии скорее всего есть. разумные формы жизни, либо вымерли либо изобрели варп-двигатель и спиздили сраный трактор из сраной глизе. Так что мы там можем найти только руины этой цивилизации, ито придется охуенно искать, время их наверняка неплохо скрыло.

Аноним Птн 15 Авг 2014 10:36:34 #493 №122352 

>>122347
О, вылез мудак, со своими фантазиями. Привет, давно тебя не было. Репетитора по физике попроси у мамки нанять.

Аноним Птн 15 Авг 2014 10:52:37 #494 №122354 

Какого цвета нейтронные звёзды?

Аноним Птн 15 Авг 2014 14:24:25 #495 №122367 

>>122350
Представь, что пришельцы прилетели на Землю через миллиард лет после вымирания человека. Найдут ли они хотя бы одно свидетельство его существования как разумного вида?

Аноним Птн 15 Авг 2014 14:30:50 #496 №122368 

>>122367
Кучи космолома в Солнечной системе.

Аноним Птн 15 Авг 2014 14:49:52 #497 №122371 

>>122368
Что, вы всерьез считаете эти скопления железно никелевых астероидов остатками рукотворных сооружений, которые космической коррозией размолотило в пыль?

Аноним Птн 15 Авг 2014 15:12:03 #498 №122373 

>>122371
>космической коррозией
Што простите? Аппараты некуда не денутся, если их не распидорасит об что-то другое.

Аноним Птн 15 Авг 2014 15:36:08 #499 №122375 

>>122371
Ты так говоришь, будто аппараты прямо таки истираются в кашу в космосе. Вангую, 95% сообщений об этом в спейсаче оставлены одним человеком. Ебаный спутник размером с автобус на гелиоцентрической - ну что ему будет? Покроется коркой говна, но внутри останутся признаки рукотворности.

Аноним Птн 15 Авг 2014 16:16:57 #500 №122376 

>>122375
>>122373
Я не видел, чтобы он говорил про истирание в кашу, тащемта. Он говорил, что инопланетяне хуй опознают что там у этих каменюк внутри, точно так же как и мы не можем сказать, что внутри у кометы Ч-Г.

Олсо, речь шла про миллиард лет, это немного другой срок. После миллиарда вообще никакого космолома не останется, все давно ебнется на землю, где уже после 100к лет рукотворность хуй отличишь, а уж тем более миллионов. А если что-то чудом уцелеет - например на гелиоцентрической орбите, да и то нет гарантии - то оно либо превратится в пыль, либо в однородный кусок говна.
> спутник размером с автобус
За сроки порядка миллиарда лет реголит метрами накипает. Фобос с Деймосом в начале своего существования выглядели совсем иначе чем сейчас (как предполагается). Автобус действительно превратится в кашу за такое время, вместе с движком и прочей хуйней, даже если отбросить внутреннюю деградацию (за счет диффузии например).

Аноним Птн 15 Авг 2014 16:20:40 #501 №122377 

>>122375
Но, вообще, не факт, конечно, что за миллиард лет аппарат просто не ебнется на что-нибудь. Исходя из логики, что природное говно, что могло ёбнуться - уже ёбнулось миллиарды лет назад, и по космосу сейчас летает то, что находится на относительно годных и синхронизированных с другим говном орбитах, а рукотворные спутники - это говно новое, чей потенциал ёбнуться на что-нибудь выше.

Аноним Птн 15 Авг 2014 16:36:59 #502 №122378 

>>122376
Фобос с Деймосом довольно низко летают, плюс неизвестны их происхождение и возраст толком, плюс у них размеры несоизмеримы с размерами спутников. Надо сравнивать с небольшими астероидами спутниковых размеров.

Откуда ж такая уверенность, что всё превращается в кашу?

Аноним Птн 15 Авг 2014 16:51:47 #503 №122382 

>>122378
(мимокрокодил)

> Откуда ж такая уверенность, что всё превращается в кашу?

КОСМИЧЕСКОЕ МАТЕРИАЛОВЕДЕНИЕ — рассматривает поведение материалов в космосе или в условиях, приближающихся к космическим. При разработке и эксплуатации материалов возникает ряд основных проблем.
Испарение материалов в глубоком вакууме ниже 10 ~3 мм рт. ст. Испарение носит избират. характер и потеря металла с границ зерен примерно на порядок превышает испарение с тела зерна. Поэтому при оценке поведения материалов в космич. вакууме нельзя руководствоваться только средними хар-ками испарения. Различно ориентированные зерна металла будут испаряться по-разному (следствие анизотропии сил межатомной связи в кристалле), что приведет к нек-рой шероховатости поверхности. В космосе сильно проявляется корпускулярная природа эффектов давления. Свободный пробег частиц составляет величину порядка неск. км. По этой причине компоненты испарения, оторвавшиеся от поверхности космич. корабля, практически больше не встретятся с этой поверхностью.

Аноним Птн 15 Авг 2014 17:24:57 #504 №122385 

>>122378
За миллиард лет в кашу превратится все что угодно, тупо из-за внутренних процессов. Это немножко не то время, чтобы можно было говорить о сохранении чего-либо. Не говоря уже о нахождении в вакууме под излучением и потоком частиц.

Аноним Птн 15 Авг 2014 17:46:59 #505 №122388 

>>122346
>>122347
Ну и кто прав?
>>122347
Хули она тогда сама себя не распидорашивает до этих самых частиц?

Аноним Птн 15 Авг 2014 17:49:21 #506 №122389 

>>122385
А циферки какие-нибудь красивые есть?

>>122382
Не совсем понятно, что цитата говорит. Железные астероиды летают себе вполне, не испарились.

кто там перекат решит запилить Аноним Птн 15 Авг 2014 17:51:31 #507 №122390 

Перепишите текст в шапке во что-то более согласованное. А то у меня анальный баттхёрт жепы от хромой стилистики первого предложения. Его, кстати, я и писал.

Аноним Птн 15 Авг 2014 19:13:23 #508 №122404 


Перекат
https://2ch.hk/spc/res/122403.html
>>122403
>>122403
>>122403

Аноним Птн 15 Авг 2014 19:49:09 #509 №122409 
1408117749454.jpg

>>122388

Суть токова, что оба правы.
Но начнём мы с того, что ты пиздоглазо ЧД - не обьект с чоткими границами и считать его "массу" довольно сложно, и вот чому: ЧД состоит из так называемой "сингулярности" - сколлапсировавшей звезды, плюс окрестности этой звезды, называемыми разиусом Шварцшильда, где сильна гроветацца настолько, что пизда штурвалу. Быдлолюди запросто называют эту окрестность как раз "ЧД". И вот что мы имеем - летит себе планетка, и тут хобана чорная дыра пиздец приехали ща всосёт. Но на самом деле куча вариантов по мере возрастания ужоса:
1) Планета летела криво и не смогла выйти на орбиту вокруг ЧД::
- плонета просто провернётся мимо ЧД, изменит свою первоначальную траекторию, но полетит дальше.
2) Вышла на орбиту ЧД::
- Крутится по орбите ЧД. Всё равно, что перебегать восьмиполосный автобан пешком. Сшибёт мимодругой планетой\звездой, но эт хуйня. Вот если пролететь над полюсом ЧД - охуярит аккреционным выхлопом, такие штуки сверху\снизу ЧД, туда лучче не лезть.
3) Подлетела совсем близко - на то расстояние, где разница гроветацци от ЧД разная на ближней и дальней стороне планеты::
- Планету распидорашивает. И закручивает по малюсенькой орбитке. Но не переживай, ей не будет грустно - там же крутится тысячи таких же ранее распидорашеных звёзд и плонет. Если повезёт, можно съебать от ЧД, проползя к полюсу и скипуть от неё на джете-выхлопе.

4) Подлетела оче близко::

Как в предыдущем пукте, но останки плонетки залетают под "горизонт событий" - вылететь отттуда уже низя, ваще никак(хотя хз ещё). Атомы плонетки так же уныло нарезают круги
вокруг сингулярности, но снаружи этого уже не видно. Масса ЧД по идее увеличивается, но по сути - изменился центр масс. То есть, теперь ещё учитывается и масса "плонетки". То, что летало снаружи ЧД теперь крутится вокруг не чисто сингулярности, а общего центра масс сингулярности+плонетки.

5) Пилот, направлявший плонету на ЧД был ассом 80го уровня, и проложил дорожку точно в центр сингулярности:

- предыдущие пункты, только теперь-таки масса плонетки поглотится ЧД по-труЪшному - сольётся с сингулярностию.


Самое важное: вероятность событий по мере устрашнению уменьшается обратно пропорционально. То есть, скорее всего, произойдёт вариант №1. Велика вероятность, что будет и №3. Если СИЛЬНО повезёт, то будет №4. Вариант №5 настолько маловероятен, что учитывать его не имеет смысла - полети планетка +- километр вправо\влево, она уже не попадёт в "яблочко", а закрутится по орбите, пусть и распидорашеная, пусть внутри ЧД, но тем не менее, не сольётся с сингулярностью.

Конечно, переход от пункта к пункту происходит потихоньку сам по себе из-за деградации\прецесии орбиты. Но мы это не рассматриваем, ибо вопрос был вроде не про это.

Аноним Птн 15 Авг 2014 21:08:30 #510 №122420 
1408122510375.jpg

>>122389
>Железные астероиды летают себе вполне, не испарились.
Ты их видон то видел? Это основательно побитые жизнью бесформенные куски говна.
Собственно с чего я всю это тему то замутил - получается что даже если в системе существовала развитая космическая цивилизация, то мы просто про нее никогда не узнаем. Все улики уничтожены за давностью лет.

Аноним Птн 15 Авг 2014 21:30:33 #511 №122424 

>>122409
> с чоткими границами
>>122409
>плонетки
Няша, ты был нетрезв?
Алсо, сингулярность = неопределённость.
Далее, уравнения Эйнштейна, тебе сюда: http://scask.ru/book_t_phis2.php?id=98
Не забудь про преобразования Лоренца, это тут: http://scask.ru/book_t_phis2.php?id=8
После изучения вышеприведённых материалов можно уже рассуждать о ЧД.

Аноним Суб 16 Авг 2014 05:25:41 #512 №122444 

>>122420
> Это основательно побитые жизнью бесформенные куски говна.
Так они и этой формы и не имели.

Аноним Суб 16 Авг 2014 12:11:14 #513 №122452 

>>115935
Если собственное время ускоряющегося объекта замедляется, то влияние внешних сил на него (если они "порционны"), - будет замедлятся. Ведь он подвергнется воздействию меньшего количества их порций за то-же время что и покоящийся объект. А значит, влияние гравитации будет ослабевать. Так ли это?

Аноним Суб 16 Авг 2014 12:57:18 #514 №122461 

>>115935
Фикс, очепяток: Если собственное время ускоряющегося объекта замедляется, то влияние внешних сил (полей, излучений) на него (если они "порционны"), будет усиленным. Ведь за то же собственное время что и у покоящегося объекта, движущийся - получит больше порций сил, излучений, полей. А значит влияние внешней гравитации на него - будет усиленным.

Аноним Суб 16 Авг 2014 15:39:03 #515 №122472 

>>122424
Что сказать-то хотел, юродивый? К чему эти ссылки, у самого слов не хватает?

новый тред Аноним Вск 17 Авг 2014 20:48:01 #516 №122591 

>>122588

comments powered by Disqus

Отзывы и предложения